Fragenkatalog von der Yvonne

Werbung
Liebe StudentInnen,
das ist mein ausgearbeiteter Fragenkatalog, das heißt – ich hab mich zwar ordentlich ins Zeug
gelegt, es kann aber sein, dass ich das eine oder andere falsch verstanden/übersetzt etc habe,
also ich übernehme keine „Haftung“ für die Richtigkeit der Angaben ;-)
Was ich euch noch erzählen möchte, ist, welche Fragen tatsächlich bei der 1ten Klausur am
01.02.05 gekommen sind, jedenfalls an welche ich mich gerade noch erinnern kann:





M-channel,
Kerne der Amygdala und Weiterleitung der Information (Wohin?),
Patient H.M. von Brenda Millner,
das Gleichgewichtsorgan für Schwerkraft, wie dieses funktioniert,
Gyri, Sulci und Strukturen einzeichen (Calcarinus, superior temporal gyrus,
Amygdala, Hypothalamus, Pons, Pyramide...),
 Feedforward Inhibition aufzeichnen
 Nociceptoren (Adaptionsgeschwindigkeit, Größe)
...
Fragenkatalog und Erläuterungen
Biologische Psychologie 1 2004/05
31.12.04 14:00
Ein paar Punkte in Hinblick auf die Klausur, die sie bitte durchlesen und dann
auch entsprechend einhalten sollten:
Ich gebe, wie vorgeschrieben 3 (schriftliche) Termine zur Prüfung für diese Vorlesung:
der 2. Termin wird etwa Mitte/Ende März sein,
Anmerkung Yvonne: ist am 14.März um 15st Uhr
der 3. Termin gegen Mitte des kommenden Semesters.
In Ausnahmefällen gebe ich auch mündliche Termine – allerdings müssen sie dazu
zumindest schon einmal bei der schriftlichen Prüfung angetreten sein (egal ob mit positivem
oder negativem Ergebniss). Nutzen sie bitte unbedingt schon den/die ersten Termine (denn
nur so haben sie im schlimmsten Fall entsprechend viele weitere Termine für
Wiederholungen). Sollten sie erst beim dritten Termin antreten wollen und dann tatsächlich
Krank (oder anderweitig verhindert) sein, werde ich keine „Ausnahme“ machen und sie
müssen auf das nächste Semester warten – also bitte auf keinen Fall zu lange hinausschieben.
Diese strikten Regelungen sind bei mehr als 200 HörerInnen einfach notwendig – tut mir leid.
Sollte die Prüfung aufgrund einer hohen Teilnehmerzahl in 2 HS stattfinden, so werden sie in
einer der beiden HS eingeteilt (steht unmittelbar vor der Prüfung per Aushang am HS).
Bei der Prüfung selbst (kennen Sie möglicherweise schon von Prof. Allesch), warten sie
bitte vor dem HS. Beim Eintreten wird dann der Studentenausweis kontrolliert und Sie
erhalten einen Aufkleber, der dann auf die Klausur geklebt wird. Zur Klausur selbst benötige
sie nur einen Kugelschreiber (meinetwegen einen in Reserve), aber keinen Taschenrechner,
keine Unterlagen, keine Schmusetierchen oder Glücksbringer (meinetwegen im Rucksack)
oder zusätzliche Papiere etc. Dieses (komische, etwas bürokratische und umständliche)
Prozedere erfolgt deshalb, weil sie nur so während der Klausur dann auch ungestört abreiten
können; andernfalls müsste ich während der Klausur herumgehen und die Ausweise
kontrollieren oder dann beim Abgeben, was sich nicht sinnvoll machen lässt.
Wenn die Fragebögen dann verteilt werden geben Sie ihren Aufkleber darauf und beginnen
mit der Arbeit). Bei meinen Klausuren, ist die Zeit im Normalfall kein sonderlich wichtiger
Faktor – das bedeutet sie kommen zeitlich sehr leicht zurecht.
Zur Teilnahme an der Klausur ist eine verbindliche Anmeldung erforderlich. Die
Anmeldelisten hängen ab etwa 2 Wochen vorher aus (im Notfall können sie sich auch per
email an Fr. Stummvoll [email protected]) anmelden. Ich erstelle nur so viele
Fragebögen, wie gültige Anmeldungen vorliegen sind (ansonsten fallen im Jahr etliche
hundert Euro für unnötige Kopien an). – deshalb können nur diejenigen antreten, die
angemeldet sind! Sollten Sie angemeldet sein und doch nicht teilnehmen, so können sie sich
(bis zur angegebenen Frist) aus der Liste wieder ausstreichen bzw. per email an Fr Stummvoll
abmelden).
Sollten Sie trotz Anmeldung nicht an der Prüfung teilnehmen, werden sie von mir (im Notfall)
keinen dieser zusätzlich angebotenen mündlichen Prüfugstermine erhalten – also wenn sie
nicht teilnehmen, dann melden Sie sich unbedingt ab (per Email an Fr. Stummvoll oder
einfach rechtzeitig ausstreichen!!!)
Fragenkatalog:
Der Großteil der Fragen (im aktualisierten ersten Teil) ist ident mit denen des Vorjahres,
wobei ich nicht unbedingt empfehle sich auf die „ausgearbeiteten“ Antworten zu verlassen die
irgendwo im Umlauf sein dürften.
Das Beste ist natürlich sich mit dem Stoff selber auseinanderzusetzen … aber das wissen sie
ja alle 
Einige, wenige neue Fragen sind eingefügt und einige (noch weniger) hab ich überhaupt
gestrichen. – Also der Großteil ist wie voriges Jahr (da heute Silvester ist könnte das natürlich
verwirren – ich meine wie bei der Vorlesung 20043/04).
Im 2. Teil dieses Kataloges, der inhaltlich in der VO noch nicht besprochen wurde, habe ich
noch nichts geädert, da ich eben noch nicht genau weiß, was ich (zeitlich) alles vortragen
kann – ein paar Bereiche werden sich da aber ändern.
Dieser Fragenkatalog ist, da die Vorlesung ja noch nicht abgeschlossen ist, somit noch „im
Entstehen“ und umfasst nicht alle möglichen Fragen, die bei der Klausur gestellt werden
können.
Wie angekündigt werde ich etwa ein Drittel bis die Hälfte der Fragen nicht diesem Katalog
entnehmen! (viel bleibt eh nicht übrig)
Alle Fragen sind, sofern es nicht gesondert erwähnt ist, relativ kurz, also in maximal 3-4
Sätzen zu beantworten. Sollte eine ausführlichere Antwort gewünscht werden, steht das
explizit in der Frage.
Fragen und/oder Antworten werden zum Teil sicherlich auch einfach umgekehrt gestellt (was
sind „Synapsen?“ oder „wie nennt man die Stellen an denen zwischen 2 Neuronen die
Information übertragen wird?“ – oder – „Wer war Franz-Josef Gall?“ bzw. „Wer war
Begründer der Phrenologie?“ ....
Viel Fragen sind auch redundant – führen also zur selben oder einer ähnlichen Antwort.
Sicherlich werden auch Begriffen durch andere gleichwertige Ausdrücke verändert (zB.
primäres visuelles Aral = V1 ...)
Bei Fragen wie etwa: „Was bedeutet [möglicherweise auf einer Graphik einzeichen], dorsal,
caudal, ventral, rostral, medial, frontal, superior, inferior, ipsilateral, contralateral, lateral ?“
werden sicherlich nicht immer alle Lage- oder Richtungsbezeichnungen gefragt, sondern nur
einzelne aus der hier angegebenen Auswahl; [möglicherweise auf einer Graphik einzeichnen]
bedeutet, dass ich möglicherweise ein Graphik angebe, auf der sie die entsprechenden
Bereiche oder Richtungen einzeichnen und beschriften sollen.
Wenn ich von Versuchen und/oder Personen spreche beziehen sich die Abgaben bzw. Fragen
immer auf die in der VO durchgesprochenen Experimente (auch wenn der/die Betreffenden
sicherlich auch eine ganze Menge anderer Studien durchgeführt haben – andernfalls müssten
sie sich ja dann immer die entsprechenden Jahreszahlen etc. merken ...)
Geschichte und Überblick:
Erläutern sie stichwortartig den Versuch und die Hauptaussage der besprochenen Studie von
Olds & Milner.
Bei diesem Experiment ist die Elektrodenstimulation im Hirn von Ratten durchgeführt: Die
Ratten konnten selbst durch einen hebeldruck die Stimulation auslösen. Dabei stimulierten
sich Tiere mit Elektroden an bestimmten Stellen „zu Tode“. Die positive Verstärkung des
handelns (Hebeldrücken) wurde auf ein „Lustzentrum“ (Pleasure Circuit) zurückgeführt, das
an dieser Stelle bei Stimulation die Ausschüttung des Neurotransmitters Dopamin veranlasst.
Was ist Gegenstand der Biologischen Psychologie?
Sie untersucht den Zusammenhang zwischen biologischen (physischen) Lebensprozessen, u.z.
aller Organe, und dem Verhalten.
Wer war Franz Josef Gall und was war sein Beitrag zu Biologischen Psychologie?
Gall war ein österreichischer Medizinstudent, der 1809 die Hypothese aufstellte, dass die
Ausbuchtungen der Schädeloberfläche mit Persönlichkeitseigenschaften zusammenhängen.
Damit hatte er die Phrenologie begründet und die Hirnforschung in der Öffentlichkeit populär
gemacht.
Wer war Paul Broca und was war sein Beitrag zu Biologischen Psychologie?
Der französische Neurologe Broca untersuchte das Gehirn eines Mannes der seine
Sprechfähigkeit nach einer Läsion verloren hatte und entdeckte das Sprach-Areal im
Temporallappen.
Wer war Wernike und was war sein Beitrag zu Biologischen Psychologie?
Wernike, ein Neurologe und Psychiater aus Deutschland, fand über Studien an läsionierten
Gehirnen neben dem Broca Areal noch ein zweites Areal, welches für Sprachproduktion und
–verständnis zuständig war. Das Wernike-Areal liegt im posterioren Part des Gyrus
temporalis.
Wer war Roger Sperry und was war sein Beitrag zu Biologischen Psychologie?
Sperry untersuchte Split-Brain-Patienten, d.h. Epileptiker, denen zur Behandlung ihrer
Krankheit der Corpus Callosum durchtrennt worden war. Die Epilepsie war zwar dadurch
gemildert, aber Informationen wurden dann nur mehr von einer Hemisphäre zur selben
weiterverarbeitet. So konnte er wesentliche Verarbeitungspfade in ipsi- und kontralaterale
Bahnen unterscheiden – in erster Linie in Bezug auf die visuelle Wahrnehmung und
Verarbeitung.
Wer war Santiago Ramon Y Cajal und was ist sein wissenschaftlicher Beitrag in den
Neurowissenschaften?
Cajal war ein Histologe (= Forschung mit mikroskopischer Untersuchung) der sich Golgi’s
Methode des „Golgi stain“ (Hirnmasse färbt sich, wenn sie in Silberchromlösung eingetaucht
wird) zunutze machte, weiterentwickelte, und damit die „neuron doctrine“ aufstellte:
Neuronen sind eigenständige Zellen, das heißt, sie hängen nicht kontinuierlich untereinander
zusammen, sondern kommunizieren miteinander über Kontaktstellen.
Was versteht man unter Phrenologie und wie stehen sie zu den entsprechenden Aussagen?
Die P untersucht die Oberflächenstruktur des Schädels und schließt daraus auf
Persönlichkeitseigenschaften, Krankheitsveranlagungen etc. Eine eigene Schädeltopographie
teilt die verschiedenen Areale der Oberfläche den Funktionen zu. Dies erwies sich jedoch als
Unfug, zumal schon die Schädeldecke nicht der Form der Cortexoberfläche entspricht.
Was versteht man unter Split-Brain Patienten und was ist die Hauptaussage, die sich aus
Studien mit ihnen ableiten lässt?
Sperry untersuchte Split-Brain-Patienten, d.h. Epileptiker, denen zur Behandlung ihrer
Krankheit der Corpus Callosum durchtrennt worden war. Die Epilepsie war zwar dadurch
gemildert, aber Informationen wurden dann nur mehr von einer Hemisphäre zur selben
weiterverarbeitet. So konnte er wesentliche Verarbeitungspfade in ipsi- und kontralaterale
Bahnen unterscheiden – in erster Linie in Bezug auf die visuelle Wahrnehmung und
Verarbeitung.
Was ist das BOLD Signal und bei welchem bildgebendem Verfahren ist das relevant?
Blood Oxygenation Level Dependent Signal analysiert die Verschiebung des Verhältnisses
des sauerstoffreichen und sauerstoffarmen Blutes, die durch die neurovaskuläre Kopplung von
Aktivierungen in den zuständigen Arealen Zustande kommt. Das unterschiedlich
Sauerstoffhaltige Blut hat unterschiedliche magnetische Eigenschaften und führt so zu
Signalveränderungen im Sekundenbereich. fMRI
Inwiefern ist der regionale cerebrale Blutfluss für die kognitive und/oder biologische
Psychologie relevant? Was kann damit untersucht werden und welche Verfahren arbeiten
direkt und/oder indirekt mit diesem Prinzip
Durch das Blut wird Sauerstoff, Glukose und andere notwendige (Nähr-)Stoffe in das Gehirn
transportiert. Bei PET, SPECT, fMRI, rcBF wird über Bildgebende Verfahren der
Sauerstoff/Glukose/radioaktive Gehalt (bei letzterer Option werden dazu vorher radioaktive
Stoffe iniiziert) in den Blutbahnen des Gehirns gemessen: Die bei bestimmten Aufgaben
aktivierten Areale haben eine erhöhte Durchblutung und somit andere Eigenschaften, die dann
gemessen werden können.
Gliederung des ZNS, Neuron und Gliazellen
Was ist unter weißer bzw. grauer Masse im Zusammenhang mit dem ZNS zu verstehen und
woraus bestehen diese?
Die weiße Masse besteht aus den Axonen und ist die Synapsenschicht des ZNS. Sie dient der
Verbindung der Neurone, welche die graue Masse, die Zellkernschicht, ausmachen.
Was sind Gyri bzw. Sulci? Nennen sie je vier Beispiele.
Gyri sind Ausfaltungen des Cortex z.B. präzentraler- und postzentraler Gyrus, Gyrus Cinguli,
Gyrus Parahippocampalis, Supramarginal Gyrus, Orbital Gyrus... Sulci sind die Vertiefungen
zwischen den Faltungen des Cortex z.B. Sulcus Calcarinus, Sulcus prä-postzentralis, Sulcus
centralis (Rolandis.), Sulcus lateralis...
Was gehört zum ZNS, was zum PNS?
Das ZNS besteht aus Cerebrum, Cerebellum, Hirnstamm und Rückenmark. Das PNS sind die
Nervenfasern in der Peripherie (im restlichen Körper: den Gliedmaßen, Organen etc.)
In welche 4 „Lappen“ (Lobes) wird das Gehirn normalerweise untergliedert?
Frontal-, Temporal-, Parietal- und Okzipitallappen
Was bedeutet, dass die rechte und die linke Großhirnhemispähre spiegelbildlich gleich sind
und stimmt dies überhaupt?
Das soll heißen, dass in beiden Hemisphären dieselben Strukturen (also entweder 2mal d.h.
auf jeder Seite oder nur einmal u.z. genau in der Mitte) vorkommen, was aber nicht 100%ig
zutrifft, zumal zumindest ihr Volumenverhältnis nicht übereinstimmt.
Aus wie vielen Nervenzellen (ganz grob geschätzt) besteht das menschliche Gehirn? Wie
verteilt sich das Verhältnis zwischen Groß und Kleinhirn?
Das Gehirn besteht aus 10^10 bis 10^11 Nervenzellen, wobei im Cerebellum, das nur 10%
des Gesamtvolumens des Gehirnes ausmacht, mehr als die Hälfte aller Neuronen sind.
Wie ist das zahlen- bzw. volumsmäßige Verhältnis von Neuronen zu Gliazellen im Gehirn?
Zahlenverhältnis ca 1:10 (Angaben von 1:3 bis 1:50) Volumenverhältnis etwa 1:1
Ein Oligodendroglia kann mehrere Axone umhüllen, zwischen den Neuronen befinden sich
aber auch noch Astrozyten... also???
Was ist das Spezifische an Nervenzellen im Gegensatz zu anderen Zellen?
Nervenzellen haben zur Informationsweiterleitung Axone – das haben andere Zellen nicht.
(Nervenzellen sind die Funktionellen Einheiten des Nervensystems und können Informationen
verarbeiten und weiterleiten.)
Was ist ein Axon?
Die efferente (wegführend) Verbindung vom Zellkörper (Soma) zu Verbindungsstellen
(Synapsen), also der Output-zuständige Teil der Zelle.
Aus welchen Teilen besteht eine Nervenzelle (ausgenommen der Zellorganellen?)
Dendriten, Soma und Axon, das über Synapsen mit anderen Zellen verbindet.
Was sind Dentriten?
Die afferente (hinführend) Verbindung von Verbindungsstellen zum Zellkörper, also der
Input-zuständige Teil der Zelle.
Wie nennt man zellkörpernahe bzw. -ferne Dentriten
Nah: Basal
Fern: Apical
Wie nennt man die Verzweigungen des Axons?
Axon Collaterale
Was sind Synapsen?
Dort wo ein Axon in Kontakt mit anderen (z.B. Muskel-, Nerven-)Zellen kommt, ist die
Synapse: Die Kontaktstelle zur Innervation weiterer Zellen.
Aus welchen Strukturen bestehen Synapsen?
Synapsen bestehen aus dem präsynaptischen Axon-Terminal mit Mitochondrien und
Vessikeln, dem Synaptischen Spalt und der Postsynapse, deren Rezeptoren die Transmitter im
Spalt binden.
Wenn Neurone aufgrund der Struktur des Dentritenbaumes klassifiziert werden, welche
Gruppen gibt es dann?
Anhand der Form gibt es Pyramidenzellen oder Stellatezellen und aufgrund der
vorhandenen/nicht vorhandenen Spines gibt es spiny und aspinous zellen.
Bzw Unipolar, bipolar, Multipolar.
Was bedeutet Convergenz?
Convergenz ist die Nachschaltung mehrerer Zellen auf eine Zelle.
Was Divergenz?
Divergenz ist die Informationsweiterleitung von einer Zelle auf mehrere Zellen.
Skizzieren sie den Schaltplan einer Feedforward Inhibition
Skizzieren sie den Schaltplan einer Feedback Inhibition
Was sind Glia-Zellen und was ist ihre Funktion?
Glia Zellen umhüllen die Axone und dienen der Erhöhung der
Informationsleitungsgeschwindigkeit, die Informationssicherung durch diese Hülle verringert
auch die benötigte Energie (da der Austausch von Ionen nur in den Ranvierschen
Schnürringen erfolgen kann) und die Feuerrate wir dadurch erhöht.
Was sind bzw. was ist die Funktion von Astrozyten, Oligodendrozyten oder Schwann-Zellen?
Diese Zellen sind Gliazellen: Schwannzellen dienen der Umhüllung (Myelinisierung) der
Axone der Nervenzellen in der Peripherie, Oligodendrozyten im ZNS. Astrozyten befinden
sich im ZNS und füllen die Zwischenräume der Nervenzellen, wo sie unter anderen
Funktionen z.B. für den Potassiumhaushalt der extrazellulären Flüssigkeit zuständig sind.
Welche Zellen sind in der Peripherie / im ZNS für die Myelenisierung zuständig?
Peripherie: Schwannzellen; ZNS: Oligodendrozyten.
Was ist der Vorteil der Myelenisierung von Nervenzellen?
schnellere(, sicherere, energiesparende und höhere) Transportrate
Was hat bedeutenden Einfluss auf die Leitungsgeschwindigkeit in Axonen?
Die Gliazellen, welche die Axone umhüllen. = Myelinschicht
Wie schnell leiten unmyelenisierte, bzw. stark myelenisierte Axone die Information grob
geschätzt weiter?
Als normale Geschwindigkeit gilt 10m/s, während stark myelenisierte Axone bis zu 120m/sec
erreichen können. So gilt für ein Axon mit einem Durchmesser von 1nanom. Unisoliert=2m/s,
50% isoliert: 4m/s, 90%isoliert: 20m/s und 98%isoliert:100m/s
Fragenkatalog letztes Jahr: 10-100m/s
Wie können die Nerven in Hinblick auf ihre Leitungsgeschwindigkeit klassifiziert werden?
unmyelenisierte und myelenisierte Nerven
Was sind die Folgen einer pathologischen Demyelenisierung?
Sensorische oder motorische Ausfälle, da das Signal den am nächsten Internodium
notwendigen Schwellenwert nicht mehr erreichen kann und das System bricht zusammen.
Wie funktioniert ein einfacher Reflex? Erläutern Sie dies bitte anhand einer selbst erstellten
Skizze.
Ein Reiz z.B. an der Hautoberfläche einbrechender spitzer Gegenstand wird von den
Sensorischen Neuronen in Signale übersetzt, welche über die Nervenfasern ins Rückenmark
gelangen. Die Information gelangt zu den Interneuronen, von welchen einige Axone ins
Gehirn langen – dort wird der Reiz als Schmerz interpretiert. Andere innervieren
Motorneuronen, welche wiederum den Muskel innervieren und dieser kontrahiert sich.
Was ist das Membran-Ruhepotenzial?
Das Membranpotenzial ist jene Spannungsdifferenz zwischen dem intra- und extrazellulären
Raum. Das Ruhepotenzial herrscht dann, wenn keine Signale von einer vorgeschalteten Zelle
hereinkommen – es wird durch ein chemisches Homöostaseprinzip des Ionenhaushaltes
immer wieder hergestellt bzw. erhalten.
Was ist ein Aktionspotenzial; wo und wodurch entsteht es? Bitte etwas ausführlicher in etwa
5-10 Sätzen erläutern und durch eine Graphik veranschaulichen.
Das Aktionspotential ist die Form der Information zur Übertragung in Nervenfasern. Es
entsteht üblicherweise am Axomhügel /hiller) und läuft entlang dem Axon zu den Synapsen.
Die Zelle erhält als Rezeptor oder über die Synapsen ihrer afferenten Verbindungen
(Dendriten) eine chemische Information, welche die Spannungsdifferenz vom
Ruhemembranpotenzial verändert: Die Zelle integriert diese Informationen(Signale) und
feuert bei einem erreichten Schwellenwert. Dabei öffnen sich die Natriumkanäle, sodass bei
einströmendem Na+ die Zelle innen zunehmend positiver wird, so lange, bis die Zelle innen
positiver ist als außen. Ab einem bestimmten Punkt schließen sich die Na Kanläle, die Kalium
Kanäle sind mittlerweile offen und Ka+ kann ausströmen d.h. die Zelle wird langsam wieder
negativer.
Wie können Aktionspotenziale Information codieren?
Aktionspotentiale codieren die Information durch die Anzahl der Spikes, d.h. durch die
Feuerrate: Ein schwacher Reiz löst viel weniger Aktionspotentiale aus, als ein starker Reiz.
Außerdem können Aktionspotentiale entweder erregender (EPSP) oder hemmender (IPSP)
Natur sein.
Was passiert, wenn ein Aktionspotenzial an den Synapsen ankommt?
Die Calziumkanäle öffnen sich, Ca+ strömt ein und die erhöhte Konzentration von Kalzium
bringt die Vessikel zur Fusion mit der Plasmenmenbran der Präsynapse. Dadurch wird der
Neurotransmitter aus den Vessikeln in den Synaptischen Spalt entlassen. Dort diffusioniert er
herum bis er an die Rezeptoren der Postsynapse bindet. Das bringt eine Spannungsänderung
der Postsynaptischen Membran und das Signal wird weitergeleitet.
Was versteht man unter einem „Spike“
Ein Spike ist ein Signal, d.h. ein Aktionspotential – die Spikes sind der Code der Feuerrate
einer Nervenzelle, die auf einen Reiz reagiert.
Wie bezeichnet man rein elektrische Übertragungsstellen im ZNS?
Gap-Junctions
Was versteht man unter einer axosomatischen, axodentritischen oder axoaxonischen Synapse?
Axosomatisch: Präsynapse eines Axons kontaktiert Soma als Postsynapse
Axodendritisch: Präsynapse eines Axons kontaktiert Dendriten als Postsynapse
Axoaxonisch: Präsynapse eines Axons kontaktiert Axon als Postsynapse
Was ist der funktionelle bzw. äußerliche Unterschied zwischen Gray´s type 1 und Gray´s type
2 Synapsen?
GT1: assymetrisch exzitatorisch d.h. die nachgeschaltete Zelle löst leichter ein
Aktionspotential aus – ihre Vessikel sind rund
GT2: inhibitorisch symmetrisch d.h. die nachgeschaltete Zelle wird in der Auslösung eines
Aktionspotentiales gehemmt – ihre Vessikel sind flach
Was passiert, wenn eine Synapse „aktiv“ ist?
Ein Aktionspotential gelangt an eine Synapse, die Vessikel fusionieren mit der Membran und
schütten den Neurotransmitter in den Synaptischen Spalt, wo er an die Rezeptoren der
Postsynapse andockt.
Was versteht man im Zusammenhang mit synaptischer Tätigkeit unter Exocytose?
Die Verschmelzung von Neurotransmitter-Vesikeln mit der präsynaptischen Membran und
Abgabe der Neurotransmitter in den synaptischen Spalt.
Also, dass Transmitter abgegeben werden. Exo= aus; Cyto=Zellkörper
Was versteht man unter EPSP und/oder IPSP, wann entsteht es und was ist die funktionelle
Bedeutung?
Das EPSP ist ein excitatorisches PSP, entsteht an einer excitatorischen Synapse und erregt die
Postsynapse, sodass das membranpotential dort positiver wird.
Das IPSP ist ein inhibitorisches PSP, entsteht an einer inhibitorischen Synapse (etwa von
einem inhibitorischen Interneuron ausgelöst) und hemmt die Postsynapse, sodass das
membranpotential dort negativer wird.
Was versteht man unter räumlicher bzw. zeitlicher Aufsummierung von EPSPs und/oder
IPSPs?
Räumliche Aufsummierung beruht auf der Tatsache, dass PSP’s stärker sind, je näher die
Synapse am Zellkörper ist. Zeitliche Aufsummierung basiert auf der Geschwindigkeit der
PSP’s, sodass die Spikefrequenz (Feuerrate) mehr oder weniger schnell zu einem
Aktionspotential in der nachgeschalteten Zelle führt. EPSPs und/oder IPSPs können wegen
ihrer Veränderung zum positiveren und/oder negativeren Membranpotential aufsummiert
werden – so, dass sie sich auch gegenseitig aufheben können.
Wird eine Zelle durch ein EPSP/IPSP leichter oder schwerer erregbar?
EPSP: excitatorisches postsynaptisches Potential: Zelle leichter erregbar
IPSP: inhibitorisches postsynaptisches Potential: Zelle schwerer erregbar
Structure of the Nervous system
Wie schwer ist etwas das Gehirn eines Erwachsenen?
1350g
Was bedeutet [möglicherweise auf einer Graphik einzeichen], dorsal, caudal, ventral, rostral
medial, frontal, superior, inferior, ipsilateral, contralateral, lateral, afferent, efferent ?
dorsal= am Rücken (oben) gelegen, caudal= Schwanzwärts bei Tieren (Cauda=Schwanz) bzw
beim Menschen Fußwärts, ventral= am Bauch (unten) gelegen, rostral= Schnauzenwärts bei
Tieren bzw beim Menschen Nasenwärts, medial= in der Mitte, frontal=Vorderseitig,
superior= darober, inferior=darunter, ipsilateral=gleichseitig,
contralateral=gegenseitig(gegenüber), lateral= seitlich, afferent= hinführend, efferent=
wegführend.
Was sind Spinalnerven und wonach werden sie untergliedert bzw. benannt?
Nervenbahnen die zwischen der Wirbelsäule in der Peripherie verlaufen.
Man unterscheidet dabei zwischen den ventralen(Motorik) und dorsal(Sensorisch) und weiters
nach zervikal, thorakal, lumbal, sakral, [Kokzygealnerven letztere nicht im Bear]- nach der
Lage der Austrittstelle.
Was versteht man unter dem somatischen peripheren Nervensystem?
Alle willentlich kontrollierbaren Muskeln werden von dem somatischen peripheren
Nervensystem innerviert: Motorische und Somatische Axone.
Was versteht man unter dem visceralen peripheren Nervensystem?
Unwillkürliches, vegetatives oder autonomes NS: Neurone die die inneren Organe,
Blutbahnen und Drüsen innervieren. Die Visceral sensorischen Axone bringen die Info über
die visceralen Funktionen ins ZNS, während viscerale Motornerven die Muskeln der glatten
Muskulatur innervieren.
Skizzieren sie einen Schnitt durch das Rückenmark, zeichnen und beschriften sie dabei die
wichtigsten Strukturen sowie die Ein- und Austrittsstellen.
Einzuzeichnen sind: Nervenfasern (weiße Masse), Zellkörper (graue Masse), Vorderwurzeln
(ventral), Hinterwurzeln (dorsal) und Laterale Wurzel, ebenso Dorsal- Lateral- und VentralColumnen, Ventral-root-Fasern und ventral Root, Dorsal-root-Ganglien, -Fasern und Root,
Spinalnerven, Spinalcanal, Umhüllende Häute: Spinal pia matter, subarachnoider Raum,
Spinal arachnoid, Spinal dura mater.
Zeichnen Sie einen Schnitt durch das Rückenmark und geben Sie an, wo die lateralen, bzw.
ventromedialen Trakte verlaufen. (bzw. bei den lateralen genauer der rubrospinale und
corticospinale).
Lateral: in der Lateralen Columne -> großer dorsaler Bereich für corticospinalen tract und
kleinerer darunter angrenzend/überlappender Bereich für Rubrospinalen tract.
Ventromedial: vom Ventralhorn abwärts über die ventrale Columne.
Wieso verändert sich das Verhältnis zwischen weißer und grauer Substanz im Rückenmark
abhängig von der „Schnittstelle“?
Die weiße Masse stellt die Nervenfasern (Leitungen) dar, die graue Substanz die
Nervenzellkörper. Da abwärts die Spinalnervenpaare wegführen, um die jeweiligen
Körperregionen zu innervieren, nehmen die Leitungen abwärts ab, während die Anzahl der
Nervenkörper rel. konstant bleibt.
Von welchen „Häuten“ wird das Rückenmark umgeben?
Spinal pia matter, subarachnoider Raum, Spinal arachnoid, Spinal dura mater.
Nenne 5 wichtige Hirnnerven, deren Funktion, Name und Nummer.
I olfactoris – Geruchswahrnehmung (sensorisch)
II opticus – Visuelle Wahrnehmung (sensorisch)
III oculomotoris – Augenbewegungen und Lidklappenbewegungen (somatisch motorisch),
Parasympatische Kontrolle der Pupillenweite (visceral motorisch)
VIII vestibulocochlearis – Hörsinn und Gleichgewicht (sensorisch)
X vagus – parasympatische kontrolle des herzens, lunge und abdominalen Organen (visceral
motorisch), visceraler Schmerz (visceral sensorisch) und bewegung der Halsmuskeln
(somatisch motorisch)
Wonach richtet sich die Nummerierung der Hinrnerven?
Nach ihrer Austrittsstelle sind sie von Rostral zu Caudal durchnumeriert
Was sind Meningen, welche gibt es und in welcher Reihenfolge von außen nach innen
kommen diese vor?
Meningen sind die Hirnhäute. Es gibt in der Reihenfolge von außen nach innen dura matter
(harte Hirnhaut), arachnoidea (Spinnwebenhaut), [dazwischen subarachnoidaler Raum, gefüllt
mit liquor cerebrospinalis], pia matter (weiche Hirnhaut)
Welche der Hirnhäute legt sich direkt den Gyri und Sulci an?
pia matter – weiche Hirnhaut
Wozu dient das Liquor cerebrospinalis?
Zur Pufferung des Gehirns, Reduktion des effektiven Gewichts
So ist das Gehirn innen und außen in diese klare, salzige Flüssigkeit eingebettet.
Wieviel Liquor wird durchschnittlich produziert und wie kommt es von den Ventrikeln zu den
Arachnoidalzotten in denen es resorbiert wird?
Das LCS wird im choroid plexus in den lateralen Ventrikeln produziert – und zwar ca. 500ml
am Tag, sodass der gesamte Liquor täglich mehrmals gewechselt wird. Es fließt von den
Ventrikeln zu einigen verbundenen, ungepaarten Leitungen am Hirnstamm und kommt durch
kleine Öffnungen neben dem Cerebellum in den subarachnoiden Raum. Dort wird es von
Blutbahnen an den „arachnoid villi“ (=Arachnoidalzotten) absorbiert.
Was versteht man unter Ventrikel?
Ventrikel sind die Hohlräume des Gehirns, die mit LCS gefüllt sind.
Wodurch kann ein Hydrocephalus entstehen und was kann dagegen unternommen werden?
Ein Hydrocephalus (Wasserkopf) entsteht, wenn die Absorbtion des LCS gestört ist: Stimmt
das Verhältnis zwischen produziertem und in den subarachnoidalen Raum abgegebenem LCS
nicht, so schwellen die Ventrikel an. Diese Störung wird behandelt, indem eine Leitung in die
geschwollenen Ventrikel gelegt wird und der überschüssige Liquor abgeleitet wird,
üblicherweise in den Verdauungstrakt (zur Ausscheidung).
Aus welchen Teilen (Ventrikel und Verbindungen dazwischen) besteht das Ventrikelsystem?
2 laterale Ventrikel, dritter und vierter Ventrikel, cerebrales Aquädukt als Verbindung
zwischen 3tem und 4ten V, sowie Spinalcanal der abwärts ins Rückenmark führt.
Wie nennt man die Füssigkeit in den Ventriken und wo wird diese produziert?
Liquor cerebrospinalis wird im choroid plexus in den lateralen Ventrikeln produziert.
Entwicklung des Nervensystems:
Aus welchen drei Bereichen entwickelt sich der Embryo?
Ektoderm, Mesoderm, Endoderm
Woraus entsteht die Neuralplatte / Neuralrohr / Neuralleiste?
Ectoderm -> Neuralplatte -> Neuralrohr -> Neuralleiste
Welche Strukturen entwickeln sich später aus dem Ektoderm, Mesoderm, Endoderm?
endoderm (Organe, viscera), mesoderm (Knochen und Muskulatur), ectoderm (NS und Haut)
Wie entsteht das Neuralrohr und was wird aus diesem?
Auf der Neuralplatte bildet sich die Neurale Rille (rostral zu caudal), die seitlichen Ränder
dieser Rille sind die Neuralen Falten, welche sich zusammenfügen, also dorsal verschließen,
und so das Neuralrohr formen. Das gesamte ZNS entwickelt sich aus den Wänden des
Neuralrohres.
Was entsteht aus dem rostralen Teil des Neuralrohres?
Im ersten Schritt der Differenzierung entwickeln sich am rostralen Ende des Neuralrohres die
3 primären Vessikel. Das gesamte Gehirn entwickelt sich aus diesen drei primären Vessiklen
des Neuralrohres.
Wann muss sich das Neuralrohr schließen?
Dieser Vorgang, Neurulation genannt, findet etwa 22 Tage nach der Befruchtung der Eizelle
statt.
Wie werden die drei primären Hirnbläschen bezeichnet und was entwickelt sich daraus?
Das rostralste Vessikel ist das Prosencehalon (Vorderhirn, forebrain), woraus sich die
sekundären Vessikel differenzieren. Diese Regionen entwickeln sich dann weiter zu
Telencephalon, Olfactorischer Bulb, Olfactorische Cups, die Cerebralen Hemisphären und das
Diencephalon.
Dahinter liegt das mesencephalon (Mittelhirn, midbrain), das zu Tectum, Tegmentum und
Cerebralem Aquädukt wird.
Caudal liegt das dritte primäre Vessikel, das rhombencephalon (Hinterhirn, hindbrain), das zu
Cerebellum, Pons und Medulla differenziert und außerdem das vierte Ventrikel enthält.
Was sind die 5 sekundären Hirnbläschen?
Sie differenzieren an beiden Seiten des Prosencephalon.2 optische Vessikel, 2
telencephalische Vessikel und das Diencephalon.
Wie nennt man die Struktur, in der die Neurone, die vom motorischen Cortex in das
Rückenmark ziehen, auf die kontralaterale Seite kreuzen?
Internal capsule (wo die Pyramidenbahnen durchkreuzen)
Welche anatomischen Veränderungen im Hirnstamm sind bei Autisten bekannt (in der LV
besprochen worden)?
Im Gehirnstamm fehlt die superiore Olive. Der Abstand zwischen den Elementen ist kleiner –
der Hirnstamm ist verkürzt!
Struktur des Cortex:
Aus welchen 6 Zellschichten besteht der Neocortex?
Molecularschicht, äußere Körnerschicht, äußere Pyramidenschicht, innere Körnerschicht,
innere Pyramidenschicht, multiforme Schicht
Welcher „???Cortex“ kann dem „Neocortex“ gegenübergestellt werden und was ist der
grundlegende Unterschied?
Allocortex hat 3 Zellschichten, Neocortex hat 6 Zellschichten
In welche 4 Bereiche (3 Cortices) lässt sich das Telencephalon unterscheiden, welche
neuroanatomischen Strukturen gehören jeweils dazu und was ist deren funktionelle Relevanz?
Telencephalon (Großhirn) nach phylogenetischer Unterscheidung:
- Striatum: (keine Oberfläche) Teil der Basalganglien (Kontrollierte Bewegungen)
- Allocortex:
o Paleocortex: Ältester Teil (Riechhirn)
o Archicortex: Hippocampus (Gedächtnisbildung) und Gyrus cinguli
(Emotionen)
- Neocortex: Jüngster und größter Teil des Telencephalons, an der Oberfläche primäre
und sekundäre Assozationsfelder
Was ist die funktionelle Bedeutung folgender Brodmann-Areale: 1,2,3,4,5,6,7,17,18,19,41,42;
1,2,3 somatosensorik (evt auch 5)
4 primäre und 6 supplementäre motorik
5, 7 Assoziationscortex
17,18,19 visuelles System
41,42 akustisches System
Zeichnen sie folgende Brodmann-Areale in ein gegeben Skizze/Graphik ein:
1,2,3,4,5,6,7,17,18,19.41.42;
1,2,3 Postzentraler Gyrus
4,6 Päzentraler Gyrus und anliegende Teile des Frontalen Lappens
5, 7 Posterior Parietal
17,18,19 Okzipitaler Lappen
41,42 superior temporaler Gyrus
In welchem Hirnlappen (Lobe) finden sie das primäre visuelle, akkustische,
somatosenorische, motorische Areal?
visuell – okzipital
akkustisch – temporal
somatosensorisch – Parietal
motorisch - Frontal
Wonach hat Brodmann die Areale untergliedert?
Brodmann unterteilte den Neocortex in eine cytoarchitecturale (Cytoarchitektur=Anordnung
der Nervenzellkörper im Gehirn, in diesem Fall im Cortex) Karte: Jede Region des Cortex mit
einer einheitlichen Cytoarchitectur bekam eine Nummer.
Markieren Sie in der Graphik folgende Bereiche: Telencephalon, Diencephalon,
Mesencephalon, Pons, Cerebellum, Myelencephalon; Prosencephalon Hirnstamm (Truncus
cerebri).
Was sind Assoziation-, Kommisuren-, Projektionsbahnen?
Assoziationsbahnen verbinden Bereiche innerhalb einer Hemisphäre (z.B. Cingulum),
Kommissuren verbinden die beiden Hemisphären (z.B. Corpus callosum, commissura
anterioris und posterioris)
Projektionsbahnen projezieren Informationen weit weg (oder leiten sie her) (z.B.
Pyramidenbahnen mot.Areal ins Rückenmark)
Welche Arterien versorgen das Gehirn?
Zwei Arterienpaare versorgen das Gehirn mit Blut: Die zwei internal carotiden Arterien und
die 2 vertebralen Arterien. Letztgenannte treten neben dem basalen Ende der Brücke
zusammen und formen die Basilare Arterie.
Welche Arterie/n ist für die Blutversorgung des Frontal-, Parietal-, Occipitallappens relevant?
Die Internal carotiden Arterien teilen sich in die
- anterior cerebralen arterien die über die anteriore kommunizierende Arterie verbunden
sind, und
- mittelcerebralen Arterien
für die Versorgung des Frontallappens, Temporallappens und für den Parietallappen.
Außerdem Versorgen sie das Zwischenhirn und die Hypophyse.
Von der Basilaren Arterie zweigen die posteriore cerebellare arterie und die Superiore
cerebellare Arterie ab, die den Okzipitallappen, Temporallappen, das Kleinhirn und den
Hirnstamm versorgen.
Wie bezeichnet man die kreisförmige Verschaltung der Arterien bei der Blutversorgung des
Gehirns?
Circulus arteriosus cerebri Willisi;
---- oder kurz: circle of Willis
Was versteht man unter Blut-Hirn-Schranke und wozu dient sie?
Die Kapillarwände der Blutversorgenden Arterien im Hirn sind für viele Stoffe nicht
permeabel – also schützende Funktion.
Nennen sie 5 verschiedenen Kerngruppen des Thalamus, woher diese die Information erhalten
und wohin sie diese projizieren (abgesehen von den corticothalamischen Feedbackschleifen).
Kerngruppe im Thalamus Von wo
Wohin
nucleus anterioris
Mammillothalamischen Tract  gyrus cinguli
nucl. mediodorsalis
substanzia nigra, ventrales
 präfrontalen Cortex
Pallidum, olfactorischer
Cortex, Amygdala
pulvinar
superioren colliculus und

pretectum
Corpus geniculatum laterale optischen Tract (Auge)
 visueller Cortex (17)
Corpus geniculatum mediale inferioren colliculus,
 auditiver Cortex (41,42)
lateralen hemniscus
ventral posterior lateraler und spinothalamischer tract,
 sensorischer Cortex
medialer Thalamus
medialer lemniscus,
trigeminal lemniscus
ventral lateraler Thalamus
Cerebellum
 motorischer Cortex (4)
Retina – lgn – vis cortex
inferiorer colliculus – mgn – aud. cortex
rückenmark – intralam. – sens. cortex
rückenm. – ventr.posteriorer nuc – sens cortex
gust. nucl – ventral posterior medial – gustator. cortex
Woher erhält und wohin projiziert der nucl. anterioris, nucl. mediodorsalis, pulvinar, corp.
gen. laterale, corp. gen. mediale, ventral posterior (lateral und medial), ventral lateral, ventral
anterior seine Informationen?
mammilarkörper – n anterioris – gyrus cinguli (- siehe Limbisches System!)
subst. nigra, amygdala, olfaktorischer cort. – n mediodorsal –präfrontal cort.
colliculus superior, pretectum – pulvinar – ?
cerebellum – ventral lateral – mot cortex
basalganglien – ventral anterior – prämotorischer cortex
rest wie vorhin
Aus welchem Thamamuskern erhält der prefrontale, premotor, motor, somstosensorische
parietale, occipitale, temporale Cortex seine hauptsächliche Information?
prefrontale --> nucl. mediodorsalis
premotor --> n. ventral anterior
motor --> ventral lateraler Thalamus
somatosensorische parietale --> ventral posterior lateraler und medialer Thalamus
occipitale --> corpus geniculatum laterale
temporale --> cg mediale
Wie bezeichnet man den Thalamuskern(bereich), der fast den ganzen Thalamus umgibt?
Nucleus Reticularis
Anatomy
Zeichnen sie in eine abgebildete Graphik folgende Strukturen ein, bzw markieren sie den
Namen, falls diese Struktur nicht zu sehen ist: (G.=gyrus; S.=Sulcus)
 Anmerkung Yvonne: keine Graphik eingefügt, siehe Bear 204-239 bzw. Skriptum
Frontal-, Parietal-, Occipital-, Temporallappen;
G. precentralis, G. postcentralis, S. centralis, Lateral Fissure, Sylvian Fissure, Fissura
longitudinalis (interhemispherica, Interhemispährischer Spalt), Superior temporal G., Insula
(Insel), G. frontalis sup./med./inf; G temporalis sup./med./inf.; S.intraparietalis, G. angularis,
S. calcarinus, Cuneus, G. parahippocampalis, Uncus; frontaler/occipitaler Pol.
primäres und/oder sekundäres sensorisches Areal, primäres und/oder sekundäres motorisches
Areal, primäres und/oder sekundäres visuelles Areal, primäres und/oder sekundäres
akkustisches Areal, Gustatorischer Cortex, Brodmann-Area 1,2,3,4,5,6,7,17,18,19.41.42;
supplementary motor area, pre-motor area, prefrontal cortex, inferiotemporal cortex,
Thalamus, Hypothalamus, Hypophyse, Tectum, Tegmentum, Colliculi superiores, Colliculi
inferiores, G.Cinguli, Balken (corpus callosum mit Rostrum, Splenium, Truncus und Genu),
Pineal Gland (Pineal Body, Zirbeldrüse) Cerebellum, Pons (Brücke), Fornix, Chiasma
Opticum, Amygdala, Hippocampus, Bulbus olfactorius, Ventrikel, cerebral Aquäduct, Nervus
opticus, Chiasma opticum, Mammillarbody, Medulla, Pyramide, Olive
Basales Forebrain, Septum. Nucl Caudatus, Putamen, Globus Pallidus, Fornix, Substantia
nigra, Red nucleus, Pontine formatio reticularis, cochlear nuclei, Raphe nuclei, Superior /
inferior Olive, Medullary Pyramid, medial lemniscus,
Spinal cord:
Dorsal horn, dorsal column, lateral horn, lateral column, ventral horn, ventral column, dorsal
root, ventral root, dorsal root ganglion, spinal nerve, spinal canal, rubrospinal tract,
corticospinal tract, spinothalamic tract
Blood supply: vertebral arteries, basilar artery, carotids, circulosus arteriosus willisi
Chemical Senses
Welche Geschmacksrichtungen kann der Mensch wahrnehmen?
vier Basisgeschmacksrichtungen: Salzig, Sauer, Süß und Bitter
evt. fünfte Richtung: Umami (=köstlich auf Japanisch) durch Glutamat
evt. sechste Richtung: Fett
Gibt es Bereiche der Zunge, die ausschließlich eine Geschmacksrichtung wahrnehmen und
wenn ja, welche und wo sind diese?
Der Grossteil der Zunge ist sensitiv für alle Grundgeschmäcker, jedoch gibt es Regionen, die
besonders auf die diversen Geschmacksrichtungen ansprechen: Die Zungenspitze ist
besonders sensibel für süße, der hintere Teil für bittere, der hintere seitliche Teil für sauere
und der vordere seitliche Teil besonders für salzige Reize. Was auf der Zunge effektiv
spezifisch auf Geschmäcker reagiert, sind die Ansammlungen an Geschmackszellen in
gebündelter Form von Geschmacksknospen auf den Geschmackspapillaren.
Über welche Strukturen funktioniert die Geschmackswahrnehmung?
Auf der Zunge befinden sich die Geschmackspapillen, an deren vertieften Rillenwänden die
Geschmacksknospen (buds) sitzen, die wiederum aus den Geschmackszellen bestehen. Eine
Geschmacksknospe hat eine Geschmackspore und Nervenfasern zur Weiterleitung der
Information.
Über welche/n Hirnnerv/en werden Geschmacksinformationen weitergeleitet, wo werden sie
verschaltet und wo werden diese im Cortex repräsentiert?
Die Zunge ist über drei Hirnnerven innerviert u.z. 7, 9 und 10. Sensorische Informationen
werden so weitergeleitet: Diese Hirnnerven treten etwas unterhalb von der Brücke am
Hirnstamm ein, wo sie vom Gustatorischen Nucleus in den Thalamus (ventral posterior
medialer nucl. des Thalamus) und von dort in den primären gustatorischen Cortex (teile des
postcentralen Gyrus, Operculum und Insula) gelangt. Dieser ist eng verschaltet mit dem
Geruchssystem und leitet die Geschmacksinformationen weiter in höhere Zentren.
Wird die Geschmacksinformation hauptsächlich ipsilateral, hauptsächlich kontralateral oder
eher bilateral im Cortex repräsentiert?
ipsilateral
Zu welchen subcorticalen Strukturen werden Geschmacksinformationen weitergeleitet und
welche funktionelle Relevanz haben diese Projektionen?
Einstieg und Bündelung der 3 Cranialnerven am Hirnstamm, weiter zum Gustatorischer
Nucleus (ist Teil des solitary nucleus in der Medulla) projeziert zu
- diversen Hirnstammregionen: Schlucken, Speichelfluss, Erbrechen, Verdauung,
Atmung
- Thalamus (VPM nucleus): projeziert zu den corticalen Gebieten (Insula) die auch mit
dem Geruchssinn verkoppelt sind
-
Hypothalamus und Teile des basalen Telencephalon: Schmackhaftigkeit,
Esstrieb/motivation
Wieviele verschiedene Geruchsempfindungen kann ein Mensch durchschnittlich
unterscheiden?
Es gibt 1000 verschiedene Rezeptorzellen für diverse Gerüche, deren Kombination ergibt die
Identifikation von 10.000 wahrnehmbaren diversen Gerüchen – Professionelle Parfumetiês
oder Whiskey-Spezialisten können sogar zwischen 100.000 Gerüchen unterscheiden.
Was sind Pheromone und wodurch werden diese wahrgenommen?
Pheromone sind sexuelle Botschaften in Gerüchen und werden über das vomeronasale Organ
wahrgenommen.
Was ist das spezielle an olfaktorischen Rezeptorzellen im Zusammenhang mit der
Regeneration von Neuronen?
Rezeptorzellen der Riechschleimhaut gehören zwar zur Neuronenklasse, sind aber erneuerbar
(d.h. wenn welche absterben dann wachsen welche nach).
Welche Zelltypen gibt es in der Riechschleimhaut?
Riech- Stütz- und Basalzellen
Erläutern sie ob einzelne Geruchsrezeptoren hoch spezifisch sind oder ob sie auf eine große
Zahl von verschiedenen Gerüchen reagieren?
Es gibt über 1000 diverse Rezeptorzellen für diverse Gerüche, die Kombination ergibt eine
durchschnittliche Identifikation von 10.000 diversen Gerüchen. Eine Zelle reagiert auf einen
Stoff, die Zellen in Kombination reagieren auf eine Kombination von Stoffen: Glomerule
bilden sich aus einer Gruppe der gleichen Rezeptorzellen, reagieren zusammen auf einen
Geruch.
Über welche Stationen verläuft die Geruchswahrnehmung ab den Rezeptorzellen?
Olfactory bulb  Olfactorischer tract  Olfactory cortex und verkoppelte temporallappen
Strukturen
parallel dazu vom Olfactorischen tract  Olfactorisches Tuberkel über den Cortex zum 
Thalamus – dort im medial dorsalen Nucleus  orbitofrontaler Cortex
Welche corticalen und subcorticalen Strukturen erhalten Informationen aus dem
olfaktorischen System?
olfactorisches tuberkel, medialdorsaler Nucleus des Thalamus, orbitofrontaler cortex und zum
olfactorischer cortex verkoppelte Strukturen im Temporalllappen (gustatorischer Cortex)
vom medialen forebrainbundle laufen bahnen des orbitofrontalen cortex in die Hirnzentren
des Hirnstammes
Inwiefern ist das Geruchssystem, in Bezug auf die Verschaltungen bis zum Thalamus, von
den anderen sensorischen Systemen unterschiedlich?
Alle anderen sensorischen Systeme passieren zuerst den Thalamus, bevor sie zum cerebralen
cortex projezieren.
Was versteht man im Zusammenhang mit der Geruchswahrnehmung unter einem
„Glomerulus“, wo gibt es dieses (topographisch) und welche Informationen laufen dort
zusammen?
Pro olfactorischen bulb gibt es etwa 2000 glomeruli, in welchen etwa je 25.000 primäre
olfactorische Axone enden und so auf etwa 100 olfactorische Neurone zweiter Ordnung
konvergent zusammengeschaltet werden. Die glomeruli sind jeweils für eine auf dem
olfactorischen epithelium weit verstreuten Anzahl an Rezeptoren zuständig, jedoch alle vom
selben Rezeptortyp. Diese topographische Anordnung ist zwischen den zwei O. bulbs
symmetrisch angeordnet (2 Glomerulus und dessen weiterleitenden Axones pro Bulb).
Visual System
Eye and retina
Was versteht man unter Makula, gelber Fleck, vovea centralis, blind spot, blinder Fleck?
Makula=gelber Fleck= zentrale, der Linse gegenüberliegende Stelle auf der Netzhaut, d.h. wo
die Lichtwellen hinprojeziert werden. Hier sind die meisten Lichtrezeptoren
fovea centralis= im Zentrum der makula, schärfstes Sehen, höchste Dichte der Zapfen (cones)
d.h. Photopischen Lichtrezeptoren (Farbsehen)
blind spot=blinder Fleck= Austrittsstelle des Sehnervs, da dort die Axone zu einem
Nervenbündel zusammenlaufen, befinden sich hier keine Lichtrezeptoren und wir können hier
nichts sehen, sind an dieser Stelle sozusagen „blind“
Was versteht man unter dem nasalen, temporalen Teil der Retina?
nasal= richtung Mitte des Gesichtes also richtung Nase (Nasenhälfte)
temporal= richtung Aussenseite des Gesichtes also richtung Ohren
An welchem Punkt tritt der Nervus Opticus aus der Retina bzw. dem Auge aus?
Etwas unterhalb und nasal zur makula, am blind spot/blinder fleck/optic disc.
Wieso verschwindet der Punkt bei Abb Box 9.1 Fig A, Bear 285 [Bild wird dargestellt]?
Weil wir den Punkt so auf unseren blinden Fleck projezieren und dort keine Lichtreize
wahrnehmen können, da keine Lichtrezeptoren vorhanden sind.
Ist bei nahen oder fernen Objekten die Linse des Auges stärker gekrümmt?
Nah: dicke Linse bzw. stark gekrümmt – starke Brechung des Lichtes
Fern: flache Linse bzw. schwach gekrümmt – schwache Brechung des Lichts
Was versteht man unter Akkomodation?
Einstellung der Linsenkrümmung zur Fern- Nah- Einstellung der Brechkraft zur Projektion
des Lichtes auf die Fovea.
Was versteht man unter Adaptation?
Einstellung Hell/Dunkel: Hell ist photopisches Sehen, während bei zunehmender Dunkelheit
das skotopische Sehen zunimmt.
[für die besessenen und besonders fleissigen: Da gibt es den Kohlrausch-Knick, Übergang
vom Zapfen zum Stäbchensehen, da wo sich die Zapfen nicht mehr weiter anpassen können,
weil’s zu dunkel ist.]
Welcher externe Faktor verändert die Pupillengröße?
Die Stärke des Lichts: Je stärker das Licht, desto enger die Pupille – ist es besonders Dunkel,
weitet sich die Pupille, um so viel von den schwachen Lichtreizen wie möglich aufzunehmen.
Wenn man mit einem hellen Licht in das linke ein Auge einer Person leuchtet, wie reagiert,
dann die Pupille des linken und die des rechten Auges?
Sie verkleinern sich beide.
Was versteht man darunter, dass der Pupillenreflex „consensual“ ist?
Dass sich beide Pupillen in ihrer Weite an Lichtreize anpassen, auch wenn der Lichtreiz nur
auf ein Auge trifft.
Aus welchen Zellschichten besteht die Retina und in welcher Reihenfolge (aus der
Lichteintrittsrichtung) liegen diese?
Es sind in der Retina etwa 55 verschiedene, hoch spezialisierte Zelltypen bekannt. Die
wichtigsten sind Ganglienzellen, Amakrinzellen, Bipolarzellen, Horizontalzellen,
Rezeptorzellen (Stäbchen/rods und Zapfen/cones. Das Licht trifft zuerst auf die
Ganglienzellen und zuletzt auf die Photorezeptoren.
Dies hängt damit zusammen, dass die Netzhaut aus der Wand des Diencephalons entsteht und
dieses wiederum aus dem Ectoderm.
Was sind Photorezeptoren und welche beiden Typen davon gibt es in der Retina? Welche
unterschiedliche Funktion haben diese beiden Rezeptortypen?
Photorezeptoren sind die Lichtempfindlichen Rezeptorzellen auf der Retina. Es gibt die
Zapfen (cones), die photopisch auf spezifische Farbreize (rot/grün/blau) reagieren, und die
Stäbchen (rods), die skotopisch auf achromatische Lichtreize (hohe Lichtsensitivität)
reagieren.
Wieviele Photorezeptoren (Gesamtanzahl, nicht Typen) gibt es beim Menschen ganz grob
geschätzt (pro Auge)?
Ganz grob geschätzt 125 Mio, genauer gesagt siehe nächste Frage.
Wie ist das Verhältnis von Rods zu Cones?
Es gibt 20mal mehr Rods (120Mio) als Cones(6Mio).
Wodurch (äußerlich und funktionell) unterscheiden sich Rods und Cones?
Rods haben die Form von Stäbchen, sind für das skotopische/achromatische Hell/Dunkel
Sehen zuständig, können sich gut an die Lichtintensivität adaptieren (sind also sehr
Lichtsensitiv) und kommen rund um die Fovea centralis vor.
Cones haben die Form von Zapfen, sind für das photopische/chromatische blau/rot/grün (also
Farb-) Sehen zuständig und kommen hauptsächlich in der Fovea centralis vor.
Was versteht man unter scotopischem/photopischem Sehen?
scotopisch= achromatisch – keine Farben, nur Hell/Dunkel
photopisch= chromatisch – Farben (RGB)
Skizzieren sie in einer Graphik die Verteilung von Rods und Cones in der Retina.
- 20 mal mehr Rods als Cones
- die meisten Cones in Fovea centralis, dort aber keine Rods
- Rods im Ring um Fovea centralis (äußerer Bereich Makula)
- Blind spot (nasal inferior zur fovea) keine Rezeptoren
- sonst rel. gleichmäßig verteilt
Wieso sieht man in der Nacht keine Farben?
Die Cones können sich nur bis zu einem gewissen Grad an Dunkelheit adaptieren
(Kohlrausch-Knick), dann übernehmen vollständig die Rods das Sehen.
Wieso sieht man in der Nacht genau fixierte Punkte sehr schlecht?
Weil die Cones in der Fovea Centralis in der Nacht quasi „nicht funktionieren“, da sie auf
sehr schwache Lichtreize (geringe Helligkeitssensitivität) nicht ansprechen, und das Bild
eines fixierten Punktes trifft genau in die Fovea Centralis – also um etwas scharf zu sehen,
muss man ein bisschen daran vorbeisehen, damit das Bild in den Ring um die Fovea trifft
(äußere Region der Makula), wo die meisten Stäbchen sind.
Erläutern Sie, was unter dem „blinden Fleck“ (im visuellen System) verstanden wird.
Der blinde Fleck ist ein natürliches skotom, das heißt, dass wir dort nichts sehen. Am blinden
Fleck (inferior nasal zur Makula) tritt der Sehnerv aus der Retina aus und die Blutbahnen ins
Auge ein, sodass dort keine Lichtrezeptoren vorhanden sind.
Zeichnen Sie ein funktionierendes Beispiel für „den blinden Fleck“ und schreiben sie dazu,
aus welcher Distanz (etwa) mit welchem Auge welches Symbol angesehen werden muss,
damit welches Symbol verschwindet.


Halte das rechte Auge zu, den Zettel etwa 30-50cm vom Gesicht weg und fixiere das
Psychologie-Symbol mit dem linken Auge. Bewege den Zettel nach links (temporal), und der
Rhombus wird verschwinden.
Liegt der blinde Fleck nasal oder temporal in der Retina?
nasal
Was ist der große Unterschied im Membranpotenzial zwischen typischen Neuronen und
Photorezeptoren?
Das Membranruhepotenzial ist –30mV, wenn Licht auf die Photorezeptoren fällt, so
Polarisieren sie auf –60mV (Hyperpolarisation).
Sie lösen kein Aktionspotential aus, können nur De- oder Hyperpolarisieren, da sie
kontinuierlich Transmitter ausschütten.
Ab wann (ab welchen Zelltypen) gibt es im visuellen System eine Informationsweiterleitung
mittels Aktionspotenzialen?
Ab den Ganglienzellen, die aber kontinuierlich feuern, nur eben einmal stärker und einmal
schwächer.
Was passiert mit dem Membranpotenzial, wenn ein Photorezeptor beleuchtet wird?
Wenn Licht auf die Photorezeptoren fällt, so Polarisieren sie auf –60mV (Hyperpolarisation).
Was versteht man unter Dunkelstrom (dark current)?
Permanent fließender Strom: Photorezeptoren schütten kontinuierlich Transmitter aus und
Ganglienzellen feuern kontinuierlich – bei Eintreffenden Reizen ändert sich lediglich die
Feuerrate.
Für welche Wellenlängen sind die verschiedenen Cones sensitiv?
Blau: +/- 430nm, Grün: +/- 530nm, Rot: +/- 560nm (jeweils die Zentren)
[in etwa Blau: 370-490, Grün: 440-590, Rot: 460-610]
Welche beiden Cone-Typen haben ein relativ großes gemeinsames farbsensitives Spektrum
und welcher Typ gehört nicht dazu?
Rot und Grün haben ein großes gemeinsames Spektrum wo blau nicht dazugehört: zB. 550nm
nur rot und grün reagieren!
Welche Farbempfindung haben wir, wenn alle drei Farbrezeptoren gleichmäßig aktiv sind?
weiß
Sind Frauen oder Männer häufiger von Farbfehlsichtigkeit (und Farbblindheit) betroffen und
warum?
Meistens sind Männer, und meistens mit einer rot-grün Fehlsichtigkeit betroffen. Das liegt
daran, dass die Gene,w elche für Rote und Grüne Pigmente verantwortlich sind, nur auf dem
X Chromosomen liegen, während die Gene für Blau auf dem Chromosom 7 sind. Wenn auf
dem einzelnen X Chromosom eines Mannes ein Defekt ist, dann kann dies nicht (wie bei
Frauen) durch das zweite X Chromosom ausgeglichen werden. Bei Fauen müssten also schon
beide X Chromosomen einen Defekt haben, damit sie tatsächlich Fehlsichtig werden.
Wenn jemand eine reduzierte Sensitivität für grün hat und Gelb „zusammenmischen“ müsste,
was würde er machen und warum?
Er würde zuviel Grün dazugeben bzw. zuwenig Rot, sein Gelb sähe also eher Grün aus, weil
er Grün nicht gleich stark wahrnimmt, wie ein Normalsichtiger – er braucht einen stärkeren
Grünreiz um dasselbe Gelb zu empfinden.
Was versteht man unter dem rezeptiven Feld einer retinalen Ganglienzelle?
Das ist jene Region auf der Retina, welche, wenn ihre Rezeptoren stimuliert werden, das
Membranpotenzial der Ganglienzelle ändert (sie gegebenenfalls zum Feuern bringt).
Zeichnen sie die Spike-Frequenz einer Off-Center-Ganglien Zelle in die gegebene Graphik
(Bear 306) ein unter erläutern sie warum diese Zelle so feuert.
Off-Center-Ganglien Zellen feuern, wenn in ihrem Zentrum ein dunkler Schatten als Reiz
eintrifft, feuern nicht, wenn der Schatten in ihrem Peripheren Feld (also ums zentrum herum)
eintrifft. Sie reagieren auf Dunkelreize im Zentrum, d.h. Depolarisieren und feuern, können
aber durch dieselben Dunkelreize im Umfeld inhibiert werden. Wenn das gesamte rezeptive
Feld also dunkel ist, dann feuert diese Zelle nur wenig, also kaum mehr, als wenn das gesamte
rezeptive Feld hell ist – wegen dieser lateralen inhibition.
Zeichnen siehe Bear 306 Graphik 9.23 und 9.24.
Zeichnen sie die Spike-Frequenz einer ON-Center-Ganglien Zelle in die gegebene Graphik
(Bear 306 – aber eine ON Zelle!) ein unter erläutern sie warum diese Zelle so feuert.
Dasselbe wie oben, nur dass Licht ins Zentrum fallen muss, damit sie feuert usw.
Welche drei Typen von retinalen Ganglienzellen gibt es und welche Informationen leiten sie
vornehmlich weiter?
Magno: große Zellen, große rezeptive Felder; Bewegungswahrnehmung, kurze Reaktion auch
auf sehr subtile Reize
Parvo: kleine Zellen, kleine rezeptive Felder, Farbwahrnehmung Rot und Grün, langsame,
tonische Aktivierung
nonM-nonP: Farbwahrnehmung Blau
Was versteht man unter einer Red-On, Green-Off (oder Green-On Red Off) Ganglienzelle?
Analog zu den Off-center bzw. On-center Ganglienzellen bezüglich heller und dunkler Reize
gibt es noch die Unterteilung der Ganglienzellen bezüglich ihrer Reaktion auf Gegenfarben
(Es gibt Rot-Grün und Blau-Gelb als Gegenfarbenpaare):
Red-On, Green-Off Ganglienzellen feuern stark, wenn in ihrem rezeptiven Zentrum ein roter
Reiz eintrifft, feuern gemäßigt, wenn der rote Reiz ihr ganzes rezeptive Feld ausfüllt, und sind
inhibiert, wenn im Zentrum zwar Rot, im peripheren Feld Grünes Licht einfällt und feuern
überhaupt nicht, wenn nur in der Peripherie Grün einfällt.
Umgekehrt bei Green-On Red-Off.
Zeichnen sie die Spike-Frequenz einer Red-On, Green-Off (oder Green-On Red Off)
Ganglien Zelle in die gegebene Graphik (Bear 309) ein unter erläutern sie warum diese Zelle
so feuert.
Siehe letzte Frage und Bear 309 Graphik 9.28.
Wenn man über geraume Zeit einen intensiv roten Gegenstand (Bild) betrachtet führt dies,
wenn man dann auf eine weiße Fläche sieht zu einem Nacheffekt; welche Farbe hat dieser und
wie kommt es zu diesem Phänomen?
Das Nachbild ist die Gegenfarbe Grün, da die rotsensitiven Zapfen ermüdet sind und dem
Mischeffekt weiß (aus Rot-Grün-Blau) nicht entgegenwirken, also die Green-On, Red-Off
Ganglienzellen nicht inhibiert werden und feuern. Lokaladaption
Wenn Sie längere Zeit einen dunklen (breiten) Ring betrachten und danach auf einen weissen
Fleck sehen, was sehen Sie und warum?
Ich sehe gewissermaßen ein negatives „Nachbild“ des dunklen Ringes auf der weißen Fläche.
Das liegt daran, dass die Stäbchen der Ganglienzellen, welche zuvor auf den Reiz des dunklen
Ringes gefeuert haben, nun ermüdet sind und ebenso die hellsensitiven Stäbchen der
Ganglienzellen, welche zuvor auf den Inneren und äußeren weißen Kontrast gefeuert haben,
ermüdet sind und die jeweils entgegengesetzt reizbaren Zentren nicht inhibieren.
Lokaladaption: reagieren nun hypersensitiv auf einen weißen Reiz
Visual System
Central visual System
Nennen sie alle Stationen (Zellen, Kreuzungen, Verschaltungen) der visuellen Wahrnehmung
von den Receptorzellen bis in den primären visuellen Cortex.
Stäbchen und Zapfen  Horizontalzellen  Bipolarzellen  Amakrinzellen 
Ganglienzellen  Sehnerv  Optisches Chiasma  Optischer Trakt  Seitlicher
Kniehöcker des Thalamus (CGL)  Optische Strahlung (radiation)  Primärer visueller
Cortex (V1)
Was versteht man unter visuellem Feld, binoculärem Feld, Hemifeld?
 visuelles Feld: gesamte räumliche Region, die mit beiden Augen gesehen wird wenn
wir geradeaus einen Punkt fixieren
 binoculäres Feld: Der Teil des visuellen Feldes, der von beiden Augen gesehen wird
(feststellbar durch abwechselndes schließen des einen und anderen Auges)
 Hemifeld: Die Hälfte des visuellen Feldes, wenn man eine vertikale Mittellinie durch
den fixierten Punkt (also in der Mitte) zieht
Wie genau werden die Nervenfasern im Chiasma opticum gekreuzt?
Jene von der nasalen Retinahälfte kreuzen kontralateral (also in die andere Hemisphäre), jene
von der temporalen Retinahälfte bleiben ipsilateral (also in derselben Hemisphäre).
Wo werden Bereiche des visuellen Feldes (zentral vs peripher, links vs rechts, oben vs unten)
im primären visuellen Areal abgebildet?
immer verkehrt, also andersherum: was zentral war ist dann peripher und umgekehrt usw.
Beschriften sie das visuelle Feld (oder die Gebiete in V1) mit den korrespondierenden
Nummern (Graphik aus Skript II Kandel wird gegeben).
Beschriften Sie die Schnittstellen, die zu den entsprechenden visuellen Einbußen führen
(Skript II (=Trepel 199 S215), bzw. Bear 308)
Ein Gegenstand, der nur vom rechten Auge, nur im rechten Gesichtesfeld, nur im linken Auge
etc. gesehen wird, wird in welcher/n Hemisphäre (primär) verarbeitet?
Jeweils in der kontralateralen Hemisphäre.
Wohin werden die Informationen aus der Retina weitergeleitet (alle relevanten Strukturen
nennen)?
(Bahnen: Optischer Nerv, Chiasma opticus, tractus opticus), CGL und superior colliculus,
(Bahn: Optische Strahlung), Okzipitaler Lappen: V1 (BA17, cortex striata) V2, V3, V4, V5,
V6
Was versteht man unter „retinotop“?
Die Verarbeitung der visuellen Information (Signale aus der Retina) erfolgt in Nervenzellen,
die ebenso wie die Rezeptoren der Retina angeordnet sind, d.h. jene Bereiche, die auf der
Retina nebeneinanderliegen, liegen z.B. auch im colliculus superioris nebeneinander.
Was versteht man unter dem „striate cortex“?
V1 (Brodmann Areal 17) ist die primäre Sehrinde und heißt wegen seines Aussehens auch
area striata oder striate cortex.
Was ist die Aufgabe der Colliculi superiores im Zusammenhang mit dem visuellen System?
Aufgaben des Colliculi Superiores sind Orientierung und visuelle Aufmerksamkeit d.h.
Augenbewegungen, realisiert durch motorische, auditive und somatosensorische
Innervierung.
Wie ist das corpus geniculatum (schichtenmäßig) aufgebaut und welche Zellarten und von
welchem Auge werden in welchen Schichten verschaltet? Am besten aber nicht
notwendigerweise erläutern sie dies mit einer einfachen Skizze.
Die 6 Schichten des LGN verarbeiten die Ganglionzelltypen P, M, nonPnonM (K) in
entsprechenden Zelltypen u.z. von der ventralen Seite her aufgelistet:
1
Contralateral Magnocellulär und Koniocellulär
2
Ipsilateral
Magnocellulär und Koniocellulär
3
Ipsilateral
Parvocellulär und Koniocellulär
4
Contralateral Parvocellulär und Koniocellulär
5
Ipsilateral
Parvocellulär und Koniocellulär
6
Contralateral Parvocellulär und Koniocellulär
Vorstellbar, sodass die Koniocellulären Schichten immer zwischen den M bzw P Schichten
liegen.
Siehe Bear 323 für Graphik.
Wie unterscheiden sich die rezeptiven Felder, die Sensibilität und die Antworteigenschaften
von M und P Zellen (im LGN)?
M-Zellen: Große Rezeptive Felder, Transiente Wellen von Aktionspotentialen, nicht
Farbempfindlich
P-Zellen: Kleine Rezeptive Felder, tonische Erhöhung der Feuerrate, Rot-Grün empfindlich
On- und Off- Center-zellen sind vermischt in allen LGN-Schichten vertreten, ansonsten
verläuft die Sensibilität retinotop zu den Parvo- und Magnozellulären Ganglionzellen.
In welchen LGN Schichten sind Zellen die für Rot/Grün, Blau/Gelb bzw Hell/Dunke sensitiv
sind?
Rot/Grün: P-Zellen, Schichten 3, 4, 5 und 6
Blau/Gelb: K-Zellen, jeweils vor jeder Schicht von 1-6 eine dünne Koniozelluläre Schicht
Hell/Dunkel: M-Zellen, Schichten 1 und 2
Woher kommen die meisten Afferenzen des LGN (ausgenommen Feedbackschleifen aus dem
Cortex)?
LGN ist das Hauptziel der Axone aus den retinalen Ganglion Zellen.
[Außerdem erhält LGN synaptische Inputs aus dem Hirnstamm, welche die Stärke der LGN
Reaktion auf visuelle Stimuli beeinflusst.]
Woher erhält der LGN neben den Afferenzen aus dem tractus opticus noch sehr starken
Input?
Abgesehen von dem Retinalen Input kommen 80% der excitatorischen Synapsen kommen aus
dem primären visuellen Cortex = Feedbackschleifen.
In welche Schichten (inkl Subschichten) kann der primäre visuelle Cortex untergliedert
werden?
I, II, III, IVA,IVB, IVCalpha, V4Cbeta, V und VI
Welche Schicht in V1 erhält Information aus dem M-System, bzw aus dem P-System und
wohin werden diese hauptsächlich weiterverschaltet?
Magnozellular  IVCalpha  IVB
Parvozellular  IVCbeta  III (Blobs und Interblobs)
Wohin sendet die Schicht 6 aus V1 den Großteil der Information?
Zurück in den CGL (Feedbackschleifen).
Was versteht man unter ocularen Dominanzsäulen (ocular dominance columns)?
Die Verteilung von CGL Axonen vom linken und vom rechten Auge sind in Schicht IV in
Streifen bzw. Säulen getrennt angeordnet, jede etwa 0,5mm breit.
Was und wo sind Blobs und was ist deren funktionelle Bedeutung?
Blobs sind in Reihen angeordnete Zellsäulen, welche direkten Input aus dem CGL erhalten.
Jedes Blob ist auf eine oculare Dominanzsäule zentriert. Zwischen den Blobs sind die
Interblobs. Diese Verteilung durchläuft die Schichten II, III, V und VI.
Blobs und Interblobs erhalten Parvozellulären Input aus IVCbeta, blobs erhalten direkten
CGL Input aus den Koniozellulären Axonen. Blobs sind wiederum auf linkes/rechtes Auge
spezialisiert während die Interblobs vermischten Input erhalten.
Beschreiben Sie etwas ausführlicher die Sensitivität, den gesamten Weg, die Schaltstellen und
die Funktion des M-channels.
Die M-Zellen im CGL, Schichten 1-Contralateral und 2-Ipsilateral sind monocular, haben
Center-surround-Rezeptive Felder, reagieren mit einer transienten Feuerrate auf visuelle
Stimulation und sind achromatisch. Die Magnozellulären Neurone projezieren in die Schicht
IVCalpha, wo sie auf sog. simple cells mit Orientierungssensitivität konvergieren, d.h. dass
sie selektiv auf Bewegungsreize in den nun zusammengeschalteten Center-SurroundRezeptive-Felder Neuronen wahrgenommene Reize reagieren, je nach dem, in welche
Richtung sich der Stimulus auf der Netzhaut orientiert. Bei der Verschaltung in die nächste
Schicht, IVB, fällt die monoculare anordnung der ocularen Dominanzsäulen zum Großteil
weg, sodass viele der Neurone dieser Schicht binoculäre rezeptive Felder aufweisen.
Außerdem sind die Zellen der IVB-Schicht Richtungsselektiv, d.h. dass die Zelle auf einen
länglichen (orientierten) Reiz nur reagiert, wenn er sich in eine bestimmte Richtung über die
Netzhaut bewegt.
Zusammenfassend lässt sich also anmerken, dass die wesentliche Funktion des M-channels
die Analyse von Objektbewegungen ist.
Beschreiben Sie etwas ausführlicher die Sensitivität, den gesamten Weg, die Schaltstellen und
die Funktion des P-channels.
Die P-Zellen im CGL, Schichten 4 und 6 – Contralateral und Schichten 3 und 5 - Ipsilateral
haben kleine, monoculare Rezeptive Felder und Rot-Grün center-surround-Gegenfarben
implementiert. Genau so projezieren sie die Information weiter in die Schicht IVCbeta,
welche sie wiederum weiterleitet in Schichten II und III. Hier befinden sich die Blob und
Interblobs. Die Interblob-Zellen werden als complexe Zellen bezeichnet, da sie On und Off
Reaktionen auf Stimuli über das gesamte Rezeptive Feld verteilt aufweisen. Sie sind meistens
binocular, wenig Farbsensitiv und hochselektiv auf Orientierung.
Zusammenfassend lässt sich anmerken, dass die wesentliche Funktion des P-channels die
Analyse von Objektkonturen ist.
Beschreiben Sie etwas ausführlicher die Sensitivität, den gesamten Weg, die Schaltstellen und
die Funktion des koinocellulären-channels.
Die K-Zellen im CGL, ventrale Schichten zu 1 und 4 – Contralateral und ventrale Schichten
zu 2 und 3 - Ipsilateral sind die Fortsetzung der nonM-nonP-Zellen der Netzhaut und in ihre
Rezeptiven Felder sind die Blau-Gelb center-surround-Gegenfarben implementiert. Genau so
projezieren sie die Information weiter in die Blobs der Schichten II und III. Die Zellen der
Blobs sind Farbsensitiv und monocolar aber nicht orientationssensitiv.
Zusammenfassend lässt sich anmerken, dass die wesentliche Funktion des K-channels
gemeinsam mit der Rot-Grün-Sensitivität des P-Channels das Farbensehen ist.
[letzten 3 Fragen siehe auch Bear 331 Grafik 10.20 und 337 Grafik 10.26]
Was versteht man unter den von Hubel und Wiesel als „simple cells“ / “complex cells“ (in
V1) bezeichneten Zellen?
Simple cells sind Neurone in der Schicht IVCalpha, auf welche mehrere, derselben Linie
folgende Center-surround-Rezeptive-Felder M-Neurone, deren ON-OFF Struktur einander
ergänzend überlappt, der Retina bzw des CGL konvergieren. Daher sind sie Längsweise (also
dieser Linie folgend) orientationssensitiv.
Complexe Zellen sind Neurone des visuellen Cortex, genauer in den Interblobs der Schichten
II und III, auf welche die Parvozellulären Informationen der Schicht IVCbeta konvergieren.
Sie haben ein hoch orientationsselektives rezeptives Feld welches nicht distinktiv On und Off
Subregionen aufweist.
Was versteht man unter Orientierungssensitiven (orientation-sensitive) Neuronen?
Ein Neuron des visuellen Systemes, mit der Eigenschaft, dass es nur auf eine beschränkte
Anzahl an Orientierungsrichtungen eines Stimulus reagiert. (siehe auch Frage M-channel
bzw. simple cells)
Was versteht man unter Richtungssensitiven (direction-sensitive) Neuronen?
Ein Neuron des visuellen Systemes, mit der Eigenschaft, dass es nur auf bestimmte
Bewegungsrichtungen eines Stimulus reagiert. (siehe auch Frage M-channel bzw. simple
cells)
Was versteht man unter einem corticalen Modul (in V1)? Wie groß ist es etwa und was (and
funktionellen Teilen) muss dabei alles inkludiert sein?
Ein corticales Modul ist ein 2x2mm Würfelausschnitt aus dem cortex striata, es enthält alle
nötigen neuralen Analysefaktoren um einen visuellen Reiz, der mit beiden Augen gesehen
wird, zu verarbeiten: 2 komplette Sets an oculären Dominanzsäulen, 16 Blobs für die
Farbwahrnehmung, und in den Interblobzellen eine Komplette Repräsentation von 180°
Orientierungssäulen.
Wie verändert sich die Orientierungssensitivität, wenn sie eine Elektrode senkrecht (oder aber
parallel) zur Oberfläche in den primären visuellen Cortex stechen?
Senkrecht: Die Orientierungssensitivität bleibt die selbe für alle durchwanderten Schichten
Parallel: Die Orientierungssensitivität kreist in Schnitten wie der Minutenzeiger einer Uhr
 das liegt and dem Aufbau des Cortex in Orientierungssäulen.
Erläutern sie etwas ausführlicher Lage, Verschaltung (Input von ...), Teilgebiete und Funktion
des dorsalen Pfades.
Der dorsale Pfad erstreckt sich vom okzipitalen cortex striata (V1) über V2 und V3 zum
parientalen Lappen, wo das MT (V5) Areal und dann das MST sowie weitere dorsale
Regionen folgen. MT erhält retinotope Informationen aus V2 und V3 sowie direkte Inputs aus
der Schicht IVB von V1, daher erhält es also seine wesentlichen Informationen aus dem Mchannel. Neurone des MT Areals haben große rezeptive Felder die richtungssensitiv auf
Bewegungen reagieren. Die Organisation in Bewegungsrichtungs-Säulen ist analog zu den
Orientierungssäulen von V1. Die Neurone des MST Areals sind auf kompliziertere
Bewegungsmuster spezialisiert (lineare, radiale oder zirkuläre Bewegung). Als Funktion
dieser Verarbeitung werden Navigation (Flussfeld), Steuerung der Augenbewegung und
Bewegungswahrnehmung angenommen.
Da dieser Pfad für die Bewegungswahrnehmung zuständig ist, wird er auch Wo-Pfad genannt.
Erläutern sie etwas ausführlicher Lage, Verschaltung (Input von ...), Teilgebiete und Funktion
des ventalen Pfades.
Der ventrale Pfad erstreckt sich vom cortex striata über V2 und V3 ventral zum temporalen
Lappen in die Areale V4 und IT. V4 Neuronen erhalten über V2 Blob und Interblob
Informationen, haben größere rezeptive Felder und sind meist orientierungs- und farbsensitiv.
IT verarbeitet diese Informationen weiter und ist neben der Wahrnehmung von Objekten auch
für visuelles Gedächtnis von Bedeutung. So gibt es hier etwa auch eine Region die für
Gesichtererkennung zuständig ist.
Da der ventrale Pfad für Objekterkennung (Form und Farbe) zuständig ist, wird er auch WasPfad genannt.
Was versteht man unter Sakkaden?
Sakkaden sind die schnellen Augensprünge, sie dienen so der Blickverfolgung von unserer
Umgebung.
Was versteht man unter sakkadischer Suppression und wie kommt diese zustande und welche
Art von Inforation genau wird dabei unterdrückt?
Unsere Augenbewegungen erfolgen nicht langsam und kontinuierlich sondern sie springen,
sodass eine sakkadische Suppression erforderlich wird, während der wir für den Bruchteil
einer Sekunde nichts sehen, da die so „verschwommene“ Information von Lichtreizen nicht
weitergeleitet wird. Die Bewegungen, welche die Augen durchführen, würden ansonsten als
Bewegungsreiz wahrgenommen.
Was ist mit einer „Großmutter-Zelle“ gemeint und wie beurteilen sie dieses „Konzept“?
Die Theorie der Großmutter-Zelle besagt, dass für jedes Objekt der realen Welt eine Zelle für
die Wahrnehmung desselben existiert. Dies ist jedoch unglaubwürdig, bedenke man, dass
dadurch besagtes Objekt, etwa meine Großmutter, schon durch geringe Veränderungen nicht
mehr erkennbar wäre (z.B. wenn sie ihre Brille nicht trägt) oder aber unglaublich viele Zellen
schon alleine wegen der verschiedenen Erscheinugnsformen, Blickwinkel etc. existieren
müssten, um nur ein Objekt zu identifizieren. Tatsächlich gibt es besagte „Großmutter-Zellen“
nicht.
Was versteht man unter „Blindsight“ und wie kann dieses Phänomen erklärt werden?
Dabei geben die Patienten bei Untersuchungen an, ein Objekt nicht zu sehen, also blind zu
sein, können aber dann mit überdurchschnittlicher Treffsicherheit die Frage beantworten, ob
es sich um das eine oder andere Objekt handele bzw nach dem Objekt greifen. Man erklärt
dieses Phänomen mit einem Corticalen Schaden etwa an V1, wobei Verbindungen über das
Tectum oder CGL zum MT Areal das Blindsehen ermöglichen.
Was versteht man unter „Prosopagnosie“?
Gesichterblindheit, der Patient kann die bekannten Personen visuell nicht mehr unterscheiden
bzw. erkennen. Dieses Phänomen tritt auf, wenn Menschen am Sulcus calcarinus eine Läsion
erlitten haben, d.h. die Zuordnung Bild-Namen ist Unterbrochen.
Welche cortikalen Bereiche sind beim Balint-Holmes Syndrom betroffen und welches
Symptom haben wir diesbezüglich besprochen?
Beim Balint-Holmes Syndrom geht eine Läsion des Parientallappens voraus (also ist
vermutlich das Areal MT bzw MST oder die Verbindungen dort betroffen). Die Patienten
leiden unter der fehlenden Orientierungsfähigkeit. Wir besprachen z.B. den Fall einer
Patientin, die nicht wusste, wie sie sich richtig (gerade) ins Bett legen sollte.
Auditory System
Aus welchen Teilen besteht das Mittelohr?
Trommelfell, Ossikel=Gehörknöchelchen (Malleus, Incus und Stapes=Steigbügel) und ovales
Fenster zur Cochlea
Aus welchen Teilen besteht das Innenohr?
Cochlea und Labyrinth (letzteres ist jedoch nicht Teil des auditiven sondern des vestibulären
Systems!)
Aus welchen Membranen und Rezeptoren besteht das cortische Organ und wie funktioniert
es?
Es besteht aus Haarsinneszellen, welche zwischen die Basilarmembran und die dünne
reticular lamina eingebettet sind, den Cortistäbchen, welche die beiden Membranen
voneinander abspreizen und diversen Stützzellen. Die Haarsinneszellen besitzen jeweils etwa
100 stereocilia an ihrer Oberfläche, welche mit ihren äußeren Spitzen an die obenaufliegende
Tectoriale Membran geheftet sind. Bei eintreffenden Schwingungen setzt sich die
Basilarmembran in Bewegung während die Tectorialmembran davon unbeeinflusst bleibt und
so die Stereocilia dazwischen hin- und hergebeugt werden, was wiederum die
Haarsinneszellen zum Feuern bringt.
Wo befindet sich das Ovale Fenster (im Zusammenhang mit der akust. Wahrnehmung) und
was ist seine Funktion?
Das ovale Fenster ist die Verbindungsstelle zwischen den Gehörknöchelchen des Mittelohres
und der Schnecke im Innenohr. Bewegt sich der letzte Gehörknöchel (der Steigbügel), so
drückt die Einwärtsbewegung das ovale Fenster nach innen und infolgedessen drückt sich die
Flüssigkeit am runden Fenster nach außen, sodass dadurch die Flüssigkeit in der Cochlea in
Schwingungen gerät und die Basilarmembran in Bewegung setzt und so die Haarsinneszellen
reizt.
Wo befindet sich das Runde Fenster (im Zusammenhang mit der akust. Wahrnehmung) und
was ist seine Funktion?
Das Runde Fenster sitzt quasi „am anderen Ende“ der Schnecke – bewegt sich also parallel
zum ovalen Fenster: Drückt das ovale Fenster nach innen, so bewegt die Welle in der
Flüssigkeit das runde Fenster nach außen usw. sodass diese hin und her Schwingung die
Basilarmembran in Bewegung setzt und so die Haarsinneszellen reizt.
In welchem Bereich der Basilarmembran sind hohe (niedrige) Töne (Schwingungen)
repräsentiert?
An der weicheren Basilarmembran der breiteren oberen Apex sind die tiefen Töne, im unteren
und schmäleren Bereich die höheren Töne.
Was ist der Unterschied in der Weiterleitung von Schallinformation des Spiralganglion
zwischen äußeren und inneren Haarsinneszellen (oder Stereozilien)?
Die inneren Haarsinneszellen sind stärker auf das Spiralganglion verschaltet: Obwohl sie viel
weniger sind als die äußeren HSZ, sind sie auf mehr als 95% der Spiralganglionneurone
verschaltet.
Was ist die funktionelle Bedeutung der äußeren/inneren Haarsinneszellen des cortischen
Organs?
Da die zahlenmäßig überlegenen äußeren Haarsinneszellen in ihrer Länge varriiern können,
haben sie einen verstärkenden bzw. abschwächenden Effekt auf die inneren Haarsinneszellen.
In welchem Sinnessystem gibt es das sogenannte Spiralganglion?
Im Auditiven System: Nervenbündel zwischen den Nervenfasern des Auditiven Nervs und
den Haarsinneszellen.
Über welche Stationen (links und/oder rechts) in der richtigen Reihenfolge (beginnend beim
Spiralganglion) werden akustische Informationen weitergeleitet?
Vom Spiralganglion über den auditorisch-vestibulären Nerv in den Hirnstamm, wo jedes
Axon verzweigt und den dorsalen cochlearen nucleus und den ventralen cochlearen nucleus
ipsilateral innerviert. Vom VCN zweigen Axone ipsi- und contralateral in die superior Olive
beider Hemisphären. Über den lateralen lemniscus innervieren deren Neurone den colliculus
inferioris im Mittelhirn. Von hier geht es weiter zum corpus geniculatum mediale des
Thalamus und weiter zum auditiven Cortex.
Wenn die akustische Weiterleitung im auditory-vestibular nerve (oder nervus
vestibulocochlearis oder 8.Hirnnerv) auf der rechten Kopfseite zerstört ist. Welche
Hemisphäre/n ist sind dann bei der verbleibenden Tonwahrnehmung (der linken Seite) aktiv?
beide
Was versteht man im Zusammenhang mit der akustischen Wahrnehmung unter Tonotopie?
Die Frequenzrepräsentation in der Cochlea entspricht Anordnung der Weiterleitung im
auditory-vestibular Nerv sowie der Repräsentation in den verarbeitenden Nuclei, dem CGM
und dem auditiven Cortex.
Wodurch wird die Frequenz, bzw. die Laustärke im akustischen System codiert?
Durch die Feuerrate der Neurone und durch die Anzahl der aktiven Neurone.
Skizzieren Sie ganz kurz die besprochene Studie von Plewnia im Zusammenhang mit
Tinnitus.
Man nahm an, dass Tinnitus mit einer irregulären aktivierung des temporalparietalen Cortex
sei und dass daher also eine transiente suppression des Symptoms durch eine transcraniale
magnetische Stimulation dieser Region erzielt werden könnte. Die Forscher applizierten also
eine solche Stimulation an div. Patienten mit chronischem Tinnitus, was das Symptom
tatsächlich reduzierte. Man schloss nun auf die Bedeutung der sekundären auditorischen
Areale für Tinnitus: Vielleicht liegt diese Bedeutung in der schlechten Anpassung der
Corticalen reorganisation.
Welche 2 Möglichkeiten der Ortung einer Schallquelle in der Horizontalen haben wir
durchgesprochen?
- internaural time delay: Die Zeitverzögerung der Informationsweiterleitung eines
Reizes, der je nach Richtung zuerst beim einen, dann beim anderen Ohr eintrifft.
- internaural intensity difference: Während das dem Schallreiz zugewandte Ohr seine
volle intensität wahrnimmt, liegt das andere Ohr zu einem gewissen Grad (je nach
Richtung des Reizes) in Schallschatten des Kopfes.
Diese zwei Differenzen werden als duplex Theorie der Geräuschortung bezeichnet und von
den Neuronen des auditiven Systems verrechnet.
Wieso kann man unter Wasser die Richtung einer Schallquelle nur schlecht (bis gar nicht)
erkennen?
Weil die Schallwellen zu dicht aufeinanderfolgend an beiden Ohren fast gleichzeitig
eintreffen und die zeitliche – zu niedrige - Differenz nicht verrechnet werden kann.
Was versteht man unter binauralen Neuronen die senstitiv auf Verzögerungen (delay times)
sind? Wie funktioniert das und in welcher Struktur sind diese lokalisiert?
Binaurale Neurone sind sensitiv auf Zeitunterschiede der eintreffenden Lautreizsignale. Sie
sind auf bilateral eintreffende Inputs spezialisiert und in den superioren Oliven lokalisiert,
sodass verrechnet wird, von welchem Ohr die Information früher eintrifft.
Was versteht man unter binauralen Neuronen die senstitiv auf Schallintensität (sound
intensity) sind? Wie funktioniert das und in welcher Struktur sind diese lokalisiert?
Binaurale Neurone sind sensitiv auf Intensitätsunterschiede der eintreffenden Lautreize. Sie
sind auf bilateral eintreffende Inputs spezialisiert und in den superioren Oliven lokalisiert,
sodass verrechnet wird, von welchem Ohr die Information stärker codiert wird.
Was versteht man im Zusammenhang mit akustischer Wahrnehmung unter EE bzw EI
Neuronen und was ist deren funktionelle Bedeutung?
Es gibt EE und EI Neurone als unterschiedliche Reizbarkeit der binauralen neurone auf
internaurale Reize. EE Neurone erreichen ihre maximale Feuerrate nur, wenn sie von beiden
Ohren gereizt werden, EI Neurone werden vom einen Ohr gereizt und vom anderen Ohr
inhibiert, sind daher besonders sensibel auf Intensitätsdifferenzen der eintreffenden
Informationen der beiden Ohren.
Wodurch können Schallquellen in Bezug auf deren räumliche Höhe lokalisiert werden?
Die Form der Ohrmuschel selbst ist relevant um die räumliche Höhe (vertikale Lage) bei der
Ortung eines Geräusches lokalisieren zu können. Die Schallreflexionen ihrer Wölbungen
werden mit der Zeitdifferenz des direkt eintreffenden Schalles verrechnet.
In welcher Cortexschicht enden die Neurone die vom MGN zum primären auditorischen
Cortex führen?
Schicht IV, dicht bepackt mit Granulazellen.
Welche beiden organisatorischen Strukturen sind beim columnaren Aufbau des prim.
akustischen Cortex berücksichtigt?
- Art der binauralen interaktion (EE vs EI)
- und charakteristische Frequenz in KHZ
Was versteht man unter Tinnitus?
Ein Geräusch (Pfeifen, Rauschen, Knattern), das auf Grund einer Neuronalen Schädigung
(häufig eine zu hohe Lautstärke, zu starker Reiz vgl. Disco, Party...) ständig wahrgenommen
wird, obwohl es in der realen Welt gar nicht präsent ist.
Vestibular System
Über welchen Hirnnerv wird die Gleichgewichtsinformation weitergeleitet (Name und
Nummer)?
cranialnerv VIII, auditory-vestibular nerv, nervus vestibulocochlearis
Was sind die Hauptstrukturen des Vestibulären Systems?
Das vestibuläre Labyrinth besteht aus den otolythischen Organen und den semicirculären
Kanälen.
Wie wird gleichmäßige Beschleunigung wahrgenommen?
Das otolytische Organ besteht aus zwei großen Kammern: dem Utrikel und der Saccule. In
jeder Kammer liegt ein Sensorisches epithelium, die macula, mit Haarsinneszellen die ihre
Cilien in eine Gelatinartige Abdeckung strecken, welche wiederum mit Otolythen (Steinchen)
bedeckt ist. Durch gleichmäßige Beschleunigung zieht die Schwerkraft die Otolythen und
damit die Gelatinartige Abdeckung in eine Richtung und die Cillien reizen die
Haarsinneszellen.
Wie werden Drehbewegungen wahrgenommen?
In den Semicirculären Kanälen liegt die Ampulla (eine Ausbuchtung), wo sich die Cupula
(gelatinartige Masse) befindet in welche die Haarsinneszellen ihre Cilia strecken. Durch
Drehbewegungen gerät die Flüssigkeit in den Kanälen, der Endolymph, in Bewegung und
bewegt damit die Gelatinartige Cupula in eine Richtung und die Cillien reizen die
Haarsinneszellen.
Wohin werden die Informationen des vestibulären Systems weitergeleitet?
Die Vestibulären Nervaxone enden auf je einer Haarzelle des Vestibulären Organs. Der 8te
Hirnnerv leitet die Information weiter in den Hirnstamm zum Lateralen vestibularen Nucleus,
von da weiter ins Cerebellum, in die Limbischen Motorneuronen und in den medialen
vestibularen nucleus, von wo die Information weiter an extraoculäre Motorneuronen und
Nackenmotorneuronen geleitet wird. D.h. es gibt keine primäre Repräsentation im Gehirn.
Somatic Sensory System
Welche Mechanorezeptoren kennen Sie?
Pacinian und Meissner’s Copuscel, Ruffini endings, Merkel’s Discs
Tragen sie in ein 2x2 Felder Schema das die Adaptionsgeschwindigkeit und die Größe der
rezeptiven Felder berücksichtigt die entsprechenden Rezeptortypen ein.
[Das Schema ähnlich wie auf S 400 oben, kann vorgegeben sein, allerdings werde ich dabei
die Bezeichnungen variieren]
Größe der rezeptiven Felder
Klein
Groß
Adaptionsgeschwindigkeit
Schnell
Meissner’s Copuscel Pacinian Copuscel
Langsam
Merkel’s Disc
Ruffini Endings
Was versteht man unter Zwei-Punkt-Diskrimination (two points discrimination) und nennen
sie einen Bereich mit hoher bzw. niedriger Diskriminationsfähigkeit.
Die 2-Punkt-Diskrimination ist die Sensitivität, mit der man zwei Punktreize auf der Haut bis
zu einem minimalen Abstand noch voneinander unterscheiden kann. Hohe DF z.B. auf dem
Zeigefinger (Fingerspitzen), den Lippen... niedrige DF z.B. auf dem Rücken, den Waden...
Wie weit (etwa) müssen zwei Punkte am Rücken (oder angegebene andere Körperstelle) bzw.
am Zeigefinger (oder angegebene andere Körperstelle) auseinander liegen, damit diese als
zwei Punkte wahrgenommen werden?
Rücken: 42mm, Zeigefinger: 3mm
Welche Neuonentypen sind für Propriozeption, Mechanorezepzion, Schmerz oder Temperatur
zuständig und wodurch unterscheiden sie sich von einander?
Im Bear habe ich nur auf S 404 die Axone gefunden...?
Sie unterscheiden sich in ihrem Durchmesser und ihrer Geschwindigkeit u.z. abnehmend
von oben nach unten aufgelistet:
Propriozeption von Skeletmuskeln: Aalpha bzw. Gruppe 1
Mechanorezeptoren der Haut: Abeta bzw. Gruppe 2
Schmerz, Temperatur: Agamma bzw. Gruppe 3
Temperatur, Schmerz, Juckreiz: C bzw. Gruppe 4
Schmerz: Nociceptoren
Temperatur: Thermorezeptoren
In welche 4 Segmente wird das Rückenmark untergliedert?
Cervical, Thoracic, Lumbar, Sacral
Was versteht man unter dem Begriff „Dermatom“?
Die Hautoberfläche, die von rechten und linken dorsalen Root eines einzelnen Spinalsegment
innerviert wird (d.h. 1:1 Korrespondenz zwischen Dermatomen und Spinalsegmenten).
Über welchen „Pfad“ (inkl Zwischen- und Endstation) wird Berührung und Vibration vom
Rezeptor zum Cortex geleitet?
dorsal column-medial lemniscal pathway: Rezeptoripsilateral durch Dorsal Root
AxoneDorsale Columne RückenmarkNucleus Dorsale Columnedurch medialer
Lemniscus in der Medullakontralateral zum Thalamus (Ventroparietaler
nucleus)Primärer somatosensorischer Cortex
-----------------sofern es sich um Berührung und Vibration im Gesicht handelt, sind die
Hirnnerven 7, 9 und 10 zuständig: Trigeminal touch pathway, welche zu ipsilateralen
trigeminalen nuclei führen (analog zum dorsal column nucleus), dann kontralateral des
Vpnucleus im Thalamus usw.
Was versteht man unter lateraler Inhibition und wozu dient sie?
Dabei inhibieren sich benachbarte Zellen gegenseitig, um Kontraste zu verstärken. Dadurch
kommt es zu einer Multiplikation der eingehenden Reizinformation.
Berechnungsbeispiel für laterale Inhibition entsprechend Bear 411 allerdings mit anderen
Zahlenwerten und Gewichtungen.
Bei einer normalen Weiterleitung wird die „Stärke“ eines Reizes einfach mit 1 multipliziert,
da immer ein Neuron auf das nächste verschaltet ist.
Bei der lateralen Inhibition gibt es jeweils zwischen diesen Verschaltungen excitatorische
Synapsen und inhibitorische Synapsen mit spezifischen Gewichtungen ihrer Fähigkeit, zu
Verstärken oder zu Hemmen.
z.B. Zellen a, b, c, d, e, f, g, h, i liegen nebeneinander und sind durch excitatorische synapsen
mit der Gewichtung *4 und inhibitorischen mit der Gewichtung *-1 jeweils mit den 2
benachbarten Zellen verschaltet.
0: Input 5
a: Input 5, erhält inhibition von –1*5 von der vorhergehenden Zelle 0 und –1*5 von b (d.h.
Inhibition –5-5=–10) sowie wird selbst 4x verstärkt (d.h. Excitation von +20), ist also ein
Output von 20-10=10
b: Input 5, dasselbe wie oben nur liegt eben b zwischen a und c
c: Input 5, inhibition b –5, inhibition d –10, excitation +20 … Output 5
d: Input 10, inhibition von c –5, inhibition von e –5, excitation +40... Output 30
e: Input 5, inhib von d –10, inhib. von f –10, excitation +20... Output 0
f: Input 10, inhib von e –5, inhib von g –5, excitation +40... Output 30
g: Input 5, inhib...
h: Input 5
i: Input 5
Aus welchen Brodmannarealen besteht der primäre somatosensorische Cortex?
Vom Centralen Sulcus rückwärts (richtung caudalem ende) 3a, 3b, 1 und 2
In welchem Brodmannareal enden die Neurone die somatosensorische Information vom
Thalamus bringen?
3a und 3b
Was ist der Unterschied in der Verarbeitung somatosensorischer Information zwischen BA 3
und 1 und 2?
3a für textur, Größe und Form von Objekten, 1 textur und 2 Größe und Form.
Was versteht man im Zusammenhang mit somatosensorischer Informationsverarbeitung unter
somatotopischer Abbildung?
Die Abbildung der Rezeptoren auf der Körperoberfläche auf entsprechenden Strukturen im
Gehirn.
Was versteht man unter dem somatosensorischen Homunkulus, warum wirkt dieser so
verzerrt?
Eine Karrikatur des corticalen Vergrößerungsfaktores: Die Menge und Wichtigkeit der
Information aus den div. Somatischen Inputs korrelliert mit der Größe der zuständigen
corticalen Areale. Wenn man gemäß dieser Größenverhältnisse einen Menschen zeichnet, so
sind vor allem die Hände, der Kopf und dort besonders die Lippen und die Zunge riesengroß
im Verhältnis zum Rest.
Zeichnen sie (beschriften sie) die somatotopische Repräsentation der rechten/linken Hand, des
Gesichtes und des Fußes in eine gegebene Graphik ein.
Was wechselt sich bei der columnaren Repräsentation in somatosensorischen Cotex (zB. der
Finger) jeweils ab?
schnell und langsam adaptierende Neurone
Wieso können Phantomschmerzen (Phantomempfindungen) der Hand häufig im Gesicht
auftreten?
Das Handareal liegt im Cortex neben dem Gesichtsareal. Daher wird die nicht mehr
vorhandene (z.B. amputierte) Hand empfunden, wenn eigentlich das Gesichtsareal (d.h. der
benachbarte Teil) aktiviert wird.
Was verändert sich im Bezug auf die corticale Repräsentation bestimmter Körperareale (z.B.
Finger) bei einer Amputation bzw. bei ausgiebigem Training?
Corticale Areale von nicht mehr benötigten Strukturen werden den benachbarten Arealen
zugeteilt, z.B. wenn ein Finger amputiert wird, erhalten die benachbarten Finger seine
Cortexareale. Die Corticale Repräsentation ist veränderbar.
Was hat der posteriore parietale Cortex mit Neglect zu tun?
Beim Neglect Syndrom wird ein Teil des Körpers oder der Welt (dann bezogen auf visuellen
Neglect) ignoriert bzw. nicht als existent wahrgenommen. Der posteriore parietale Cortex ist
von essentieller Bedeutung für Wahrnehmung und Interpretation von räumlichen
Beziehungen, Körperbewusstsein und das Lernen von koordinativen Aufgaben. Dabei spielen
Verschaltungen somatosensorischer mit anderen sensorischen, wie eben visuellen,
Informationen eine große Rolle (dieser Cortexbereich zählt ja auch zum Assoziationscortex).
Welche Krankheitsbilder können bei Schädigungen des posterioren parietale Cortex
entstehen?
Agnosia: Die Unfähigkeit Objekte zu erkennen obwohl die simplen sensorischen Fähigkeiten
als intakt erscheinen. z.B astereognosia
Neglect Syndrom: siehe oben, z.B. Hemineglect: nur die Hälfte der Welt „existiert“ für den
Patienten: nur jeweils die Hälfte wird wahrgenommen, behandelt, wiedergegeben etc.
Pain
Wie werden die Schmerzrezeptoren bezeichnet?
Nociceptoren
Welche Nervenfasertypen werden in der Schmerzweiterleitung verwendet und welche davon
leiten den schnellen Schmerz, welche den dumpfen?
Der schnelle „erste“ Schmerz wird von den Agamma Fasern, der dumpfe und länger fühlbare
Schmerz von den langsameren C Fasern weitergeleitet.
Auf welche Art von Reizung reagieren Nociceptoren?
Sie werden von Stimuli aktiviert, welche möglicherweise das Gewebe verletzen könnten:
Starke mechanische Stimulierung, Extreme Temperaturen, Stauerstoffdeprivation und
Kontakt mit speziellen Chemikalien.
Welche Neurotransmitter sind bei der Schmerzweiterleitung relevant?
Glutamat und Substanz P.
Welcher Bereich spielt für die Schmerzmodulation eine entscheidende Rolle?
Rückenmark: Die Inhibition im dorsalen Horn durch Interneurone (afferente Regulation)
Periaqueductal gray matter des Mittelhirns: die Neurone projizieren in die Medulla (raphe
nuclei) und weiter in das dorsale Horn, wo sie die Aktivität der Nocizeptoren unterdrücken
(descendente Regulation)
Stellen sie die Schmerz und die Berührungsweiterleitung kurz gegenüber.
- Berührungsrezeptoren haben spezielle Strukturen in der Haut während
Schmerzwahrnehmung nur freie Nervenendungen aufweist
- Berührungsaxone Abeta sind dick, stark myälinisiert und daher schnell während
Nocicepionsaxone dünn, wenig (Agamma) oder gar nicht (C) myälinisiert und daher
langsam sind
- Abeta Axone enden tief im Dorsalen Horn der Wirbelsäule, Agamma und C Fasern
teilen sich, verlaufen in der Lissauer Zone und enden in der Substantia gelatinosa
- Die Substanzen und die Übertragung ins Gehirn sind unterschiedlich
Über welchen „Pfad“ (inkl Zwischen- und Endstation) wird Schmerz vom Rezeptor zum
Cortex geleitet?
NocirezeptorDorsal-root-GanglionEingang Dorsale ColumneWeiterleitung ventral
über Spinothalamischen TraktMedullaThalamus intralaminar und ventralparietale
NucleiPrimärer somatosensorischer Cortex
Erläutern sie die Schmerzregulation entsprechend der „Gate theory of Pain“ anhand der
Graphik (Bear 430).
Neurone des Dorsalen Hornes, welche ein Axon in den spinothalamischen Trakt projezieren,
werden von breiten sensorischen Axonen und unmyälinisierten Nocizeptor-Axonen innerviert.
Außerdem sind sie auch über ein Interneuron inhibiert, und das wiederum wird vom
sensorischen Axon excitatorisch gereizt und vom Schmerz-Axon inhibiert. So maximiert die
Aktivität des Schmerzaxons alleine die Feuerrate des Projektionsneurons. Wenn das große
mechanoreceptive Axon aber gleichzeitig feuert, aktiviert dies das Interneuron und
unterdrückt die nociceptiven Signale.
Was ist der Beitrag von Melzack & Wall zur Biologischen Psychologie?
Sie stellten die Gate theorie of Pain auf, mit der das Phänomen der Unterdrückung von
Schmerz durch (schmerzhafte) Reizung benachbarter Gebiete erklärt werden konnte.
Warum wird der Schmerz reduziert wahrgenommen, wenn man unmittelbar neben der
schmerzenden Stelle reibt?
siehe 2 vorhergehende Fragen
Welche Strukturen (in welcher Reihenfolge) modulieren absteigend (descending) die
Schmerzwahrnehmung und wie?
Periaqueductal gray matter des Mittelhirns: die Neurone projezieren mit dem
Neurotransmitter serotonin in die Medulla (raphe nuclei) und diese Neurone leiten die
Information weiter in das entsprechende dorsale Horn, wo die Aktivität der Nocizeptoren
unterdrücken, d.h. inhibiert wird. (descendente Regulation)
Welche Thermorezeptoren besitzt der Mensch?
Wärmerezeptoren und Kälterezeptoren
Auf welche Temperaturen reagieren Kälte- bzw. Wärmerezeptoren? (ganz grob die Grad in
C°) angeben
Wärmerezeptoren von 30°C bis 50°C (höchste Empfindung bei 45°C)
Kälterezeptoren von 5°C bis über 45°C und wieder ab fast 50°C wo Wärmerezeptoren nicht
mehr feuern (höchste Empfindung bei 25°)
Movement:
Welche beiden verschiedenen Muskelfasertypen werden unterschieden?
Gestreifte (Kardial-automatisch und Skeletal-willentlich),
und glatte (ANS innerviert) Muskulatur
Nennen Sie die drei Quellen aus denen Alpha Motoneurone ihre Information erhalten können.
Input aus Spinalen interneurone, Sensorische Inputs aus Muskelspindeln und Input aus
übergeordneten Motorneuronen im Gehirn
Welcher Neurotransmitter wird von Alpha Motoneuronen verwendet (ausgeschüttet)?
Acetylcholin
Was ist die Aufgabe von Alpha Motoneuronen, Ia sensory neurons, gamma Motoneuronen?
alpha MN: Kraftgeneration im Muskel: Kontraktion (Muskelverkürzung)
Ia sensorische neurone: Information, ob Muskel angespannt ist oder nicht : Propriozeption
gamma Motoneurone: Kontraktion der Muskelspindelpole, sodass sie trotz der Kontraktion
(durch alphaMN ausgelöst) gespannt bleiben (also nicht lose herabhängen)
Wann, worauf und wie reagieren Muskelspindeln?
Wenn alphaMN den Muskel zum kontrahieren bringen, so dienen die Muskelspindeln der
proprioception dieser Veränderung: Ia Axone leiten diese Information weiter, etwa an die
gamma Motoneurone.
Erläutern sie anhand einer gegebenen Graphik (etwa 13.17 S 454 ohne eingezeichnete
Neurone) den Kniereflex und zeichnen Sie die nötigen exzitat. und inhibt. Neurone ein.
Ein Hammer, der auf den Quadrizeps am Knie trifft, zieht den Muskel wie ein Gewicht in die
Länge. Dadurch werden die Muskelspindeln gestreckt. Dieses Strecken depolarisiert das
innervierende Ia axon, das nun im Rückenmark feuert und dort das innervierte Alpha motor
neuron depolarisiert. Dieses feuert nun ebenfals, u.z. zurück in den Muskel, sodass sich dieser
kontraktiert (verkürzt).
Erläutern sie anhand einer gegebenen Graphik (etwa 13.23 S 458 ohne eingezeichnete
Neurone) eine Beugebewegung (mit Flexor und Extensor) und zeichnen Sie die nötigen
exzitat. und inhibt. Neurone ein.
Das alpha MN des Flexors erhält die Information von einem übergeordneten Motorneuron
und feuert, sodass sich der Muskel kontrahiert. Dies führt zu einer Dehnung des
antagonistischen Extensormuskels. Die Muskelspindeln des Flexors feuern über ein Ia Axon
die Rückmeldung ins Rückenmark. Dort kontrollieren sie nicht nur die Feuerrate des Flexor
alpha Motorneuron sondern inhibieren zudem über ein Inhibatorisches Interneuron die
Reaktion des alpha MN des antagonistischen Muskels, sodass sich dieser nicht ebenfalls
kontrahiert sondern extensiert (relaxiert).
Skizzieren Sie die Verschaltung der reziproken Hemmung von Flexor und Extensor unter
Einfluss der Information der Muskelspindeln. (Die Graphik soll den Input der Muskelspineln
und den Output an Flexor und Extensor sowie möglicherweise nötige Interneurone beinhalten
und entsprechend beschriftet (auch exzitatorisch und inhibitorisch) sein).
Das alpha MN des Flexors feuert, sodass sich der Muskel kontrahiert. Dies führt zu einer
Dehnung des antagonistischen Extensormuskels. Die Muskelspindeln des Extensor feuern
über ein Ia Axon die Rückmeldung ins Rückenmark, wo sie das Motorneuron innervieren. Die
Muskelspindeln des Flexors feuern ebenfalls über ein Ia Axon die Rückmeldung ins
Rückenmark. Dort kontrollieren sie nicht nur die Feuerrate des Flexor alpha Motorneuron
sondern inhibieren zudem über ein Inhibatorisches Interneuron die Reaktion des alpha MN
des antagonistischen Muskels, sodass sich dieser nicht ebenfalls kontrahiert sondern extensiert
(relaxiert).
Erklären sie ganz kurz, was Myasthenia Gravis / Amyotopische Lateralsklerose ist und
wodurch dies entsteht.
Myasthenia Gravis: Mg ist eine Autoimmunerkrankung, bei der Antikörper die nikotinergen
Rezeptoren der neuromuskulären Synapsen angreifen. ACH kann nicht mehr wirken, d.h.
Ialpha MN schütten zwar ACH aus, aber die nikotinergen Rezeptoren sind defekt, sodass die
Weiterleitung der AP’s in die Muskeln nicht mehr funktioniert. Dies kann zu einer
Atemlähmung führen.
Amyotopische Lateralsklerose: Degeneration der alpha MN und Beeinträchtigung der
Neurone im motorischen Cortex, was also einen Informationsstop spätestens im Rückenmark
zur Folge hat. Als Ursachen sind bisher genetische Defekte der Abbauenzyme superoxide
dismutase, was eine Zellschädigung durch die superoxidradikale bedeutet, und Excitotoxidität
– wobei Motorneurone wegen Überstimulierung mit Glutamat absterben.
Welche beiden großen Gruppen von absteigenden Bahnen (tracts) kennen sie?
Laterale Pfade: Corticospinaler Tract und Rubrospinaler Tract
Ventromediale Pfade: Vestibulospinal, Tectospinal, Pontine Reticulospinaler Pfad und
Medullarer reticulospinaler Pfad
Zeichnen Sie in einen Querschnitt des Rückenmarks die lateralen, die ventromedialen, den
corticospinalen, den Rubrospinalen tract ein.
Siehe oben, wobei die ventromedialen in den ventralen columnen sind, die lateralen in der
lateralen columne
Was sind die Pyramidenbahnen, welche Funktion haben sie und wie ist deren Verlauf?
Die Pyramidenbahnen sind die wichtigsten Corticospinalen tracts und besonders für
feinmotorische Fähigkeiten von Bedeutung. Sie Entspringen am primären motorischen Cortex
(präzentraler Sulcus), durchwandern die internal capsule, am telencephalon und thalamus
vorbei, mit dem cerebralen peduncel in die Pons und werden an der Basis der Medulla zu
einem Tract zusammengefasst. Bei dem Übergang von Medulla zum Rückenmark kreuzen sie
und formen den lateralen corticospinalen Tract. Die Axone innervieren die motorneurone in
der dorsolateralen region der ventralen Hörner (distale Muskulatur, Flexoren).
Wozu kann eine (elektrische) Stimulation des motorischen Cortex führen?
zu Bewegungen
Welche „motorischen Cortexareale“ kennen Sie, und wo liegen diese (zB Brodmanareale oder
Gyrus/Sulcus)?
BA 4 und 6 sind der Motorcortex, stimmen aber nicht mit der Unterteilung des PMA und
SMA überein. M1 als der Primäre motor cortex macht den Präcentralen Gyrus, also BA4, aus.
Das Prämotorische Areal PMA liegt in der lateralen Region von BA6 und das Supplementäre
motorische Areal SMA liegt in der medialen region von BA6.
Was sind die Unterschiede zwischen PMA und SMA?
Sie haben ähnliche Funktionen für verschiedene Muskelgruppen:
 SMA sendet die Axone in die distalen (äußere) Motorischen Einheiten u.z. direkt,
 PMA Axone verbinden den Cortex in erster Linie mit reticulospinalen Neuronen, die
die proximalen (innere) Motorischen Einheiten innervieren.
Zeichnen sie (beschriften sie) die Repräsentation der rechten/linken Hand, des Gesichtes und
des Fußes in primären motorischen Cortex in eine gegebene Graphik ein.
Dabei ist zu beachten, dass die rechte Körperhälfte auf der linken und die linke KH auf der
rechten Hemisphäre repräsentiert ist. Einzeichnen/merken, indem man sich die Reihenfolge
der Repräsentation vom Gyrus vorwärts (rostral) einprägt:
Fuß, Knie, Hüfte, Schulter, Ellbogen, Arm, Hand, Finger (vom kleinen bis zum Daumen),
Nacken, Augenbrauen, Augenlid und –körper, Gesicht, Lippen, Kinn, Zunge, Schlucken
Welche Neurone sind (entsprechend E.Evarts) aktiv, wenn auf eine vorbestimmte Reaktion
gewartet wird (also zwischen Instruction Stimulus und Trigger Stimulus)?
Zellen des SMA auf beiden Hemisphären
Welche Strukturen gehören zu den Basalganglien?
Nucleus caudatus, Putamen (bilden gemeinsam das Striatum), Globus pallidus,
Subthalamic nucleus und Substantia nigra
Was ist die Hauptfunktion der Basalganglien?
Selektion und Initiierung von willentlichen Bewegungen.
Woher erhalten die Basalganglien (hauptsächlich) die Information und wohin wird die
weitergleitet?
Sie erhalten Information aus dem cerebralen Cortex (frontal, prefrontal und parietal) und
leiten sie durch den ventralen lateralen Nucleus des Thalamus zurück in den Cortex,
insbesondere SMA.
Eine Schädigung der Basalganglien wird zu welchen Effekten führen?
Bewegungsstörungen: Bewegungsexzess oder Bewegungsarmut bzw. Störungen bei der
willentlichen Initiierung von Bewegungen.
Welche Strukturen sind bei Parkinson/Chorea Huntington geschädigt?
Parkinson: Degeneration substantia Nigra – Störung der ausreichenden Dopaminausschüttung
Chorea Huntington: Verlust von Neuronen im caudate Nucleus, Putamen und Globus Pallidus
mit zusätzlichem Zellverlust im Cerebralen Cortex und sonst wo (kann nix für die
Formulierung, das steht so im Bear...)
Wo kommen die sogenannten „Betz-Zellen“ vor?
Große Pyramidenzellen der Schicht V im Primären Motorareal M1
Woher erhält der primäre motorische Cortex seine Hauptinformation?
Hauptinformation aus anderen corticalen Areale: BA 6 anterior und 3,1 2 posterior
(und aus dem VL nucleus des Thalamus Info aus den Basalganglien)
Wie ist cortical die Richtung einer Bewegung repräsentiert?
Die Bewegungsrichtung errechnet sich aus der Summe der bewegungsspezifisch feuernden
Neurone in M1.
Was ist die Hauptfunktion des Cerebellums?
Zeitliche Abfolge von Bewegungen
In welche Bereiche lässt sich das Kleinhirn untergliedern?
Pontocerebellum, Spinocerebellum, Vestibulocerebellum
Die Vermis in der Mitte und die beiden Hemisphären seitlich davon.
Welche Art von Tremor tritt bei Parkinsonpatienten / Pat. mit Kleinhirnschädigung auf?
Parkinson: Resting Tremor
Läsion Cerebellum: Intention Tremor
Woher erhält das Kleinhirn die Information und wohin wird diese weitergeleitet?
Vom Cortex über die Pontine nuclei in der Pons ins Cerebellum und zurück über den
vetrallateralen Nucleus des Thalamus in den Motorcortex M1.
Führen Schädigungen des Kleinhirns/der M1 im motorischen Cortex zu ipsilateralen oder
contralateralen Ausfällen?
Ich hätte gemeint beides...?
Tragen sie in die Graphik (14.8 S 491) alle fehlenden Bezeichnungen / Verbindungen ein.
...eine zusammenfassende Graphik der motorischen Verarbeitung und Weiterleitung vom
Cortex über die div. Bahnen in’s Rückenmark
Chemical controll of Behavior
Was sind die Hauptfunktionen des Hypothalamus?
Homöostase (Temperatur, Flüssigkeit, Bluutvolumen/druck, Salzhaushalt,
Sauerstoff/Glukosehaushalt...) durch:
Humorale Antwort, visceromotorische Antwort und somatisch-motorische Antwort
Welche drei Bereiche (entsprechend der LV) wurden im Hypothalamus untergliedert?
Lateraler medialer und periventricularer Hypothalamus
Nennen sie drei spezifische Hypothalamuskerne und deren funktionelle Bedeutung.
Lateral hypothalamische Region: Motivale Bewegung
Nucleus suprachiasmaticus: täglicher Hell-Dunkel Rhythmus (zirkadiane Rythmik)
Nucl Supraopticus & nucl paraventricularis : Flüssigkeitshaushalt
Mammilarkörper: Selbsterhaltung, Gedächtnisbildung, Reproduktion
mittlere Kerngruppe: Regulation der Nahrungsaufnahme
[Neurosecretorische Neuronen: Aufmerksamkeit (Verbindung zur Hypophyse)]
Aus welchen beiden Teilen besteht die Hypophyse?
Adeno- (Vorderlappen) und Neurohypophyse (Hinterlappen)
Welche Hormone werden von der Neurohypophyse ausgeschüttet und was ist deren Wirkung?
Sie sektretiert Oxytocin, das vor allem bei der Geburt und bei der Milchproduktion von
Bedeutung ist, und Vasopressin, das antidiuretisch (Niere) wirkt.
Durch welches spezifisches System wird die Information des Hypothalamus zur
Adenohypophyse weitergeleitet?
Hypothalamo-Hypophysen-Portal-Kreislauf (Hormonale Kommunikation über das
Zwischengeflecht Eminentia mediana = Kapillarengeflecht im mittleren HPBereich)
Welcher Teil der Hypophyse erhält die Information von paravozellulären/magnozellulären
Teilen des Hypothalamus?
Magnocelluläre neurosekretorische ZellenPosteriorer Hypophysenlappen
Parvocelluläre neurosekretorische ZellenAnteriorer Hypophysenlappen
Was versteht man unter Releasing/Inhibiting Hormonen, wo werden diese ausgeschüttet und
wie erreichen sie ihre direkten Zielorgane?
Sie werden von den parvocellulären neurosecretorischen Neuronen des Hypothalamus in die
hypothalamo-hypophysen portalcirculation im Voderlappen der Hypophyse ausgeschüttet.
Releasing Hormone stimulieren und Inhibiting Hormone inhibieren in der Hypophyse die
Produktion von Hormonen , welche dann über das Blut zu den Zielorgangen gelangen.
Nennen sie je zwei Hormone des Hypophysenvorder- und des Hypophysenhinterlappens.
Hinterlappen: Oxytocin und Vasopressin
Vorderlappen: Prolaktin, ACTH
Was ist die Aufgabe der Zirbeldrüse?
Melatoninproduktion
Stellen sie sympathisches und parasympathisches NS gegenüber: Austrittstellen, Transmitter,
Länge der prä und postganlionären Axone, Funktion
sympathisches Nervensystem
parasympathisches Nervensystem
Austrittsstellen thoracic und lumbarsegmente der
Hirnstamm und sacrale Segmente
Wirbelsäule
der Wirbelsäule
Transmitter
prägangl. AcH, postgangl. NE
Acetylcholin
L. Axone
kurz
lang
präganglionär
L. Axone
lang
kurz
postganglionär
Funktion
schnelle Reaktionen zur Erhaltung der Lang-Zeit-Wirkung als Erhaltung
Unversehrtheit unter Beeinträchtigung der grundsätzlichen
der langzeitlich relevanten Prozesse
Körperfunktionen, in etwa
z.B. Reaktionen aus Angst 
Verdauung, Wachstum,
Mobilisierung zur Flucht etc.
Immunantwort, Energiereserven
Welche 4 diffusen modulatorischen Systeme haben wir besprochen (nur die Namen)?
Cholinerges, Dopaminerges, Noradrenerges und Serotonerges System
Was ist/sind der Ursprung des noradrenergen diffusen modulatorischen Systems?
Locus Coeruleus
Was ist/sind der Ursprung des serotonergen diffusen modulatorischen Systems?
Raphe Nuclei
Was ist/sind der Ursprung des dopaminergen diffusen modulatorischen Systems?
Substantia nigra und ventral-tegmentale Region
Was ist/sind der Ursprung des cholinergen diffusen modulatorischen Systems?
Pontomesencephalotegmentaler Complex
Ab hier sind noch die Fragen vorläufig und es werden nur diejenigen zur Klausur kommen,
die in der letzten Einheit auch tatsächlich besprochen werden (Es kommen also keine weiteren
dazu – außer eben welcher, die vom Fragenkatalog wie Eingangs erwähnt abweichen).
Prof. hat gesagt (bei der letzten Vorlesung), dass demnach Sprache, Lateralisation und
Geschlechtsunterschiede nicht gefragt würden.
Emotion
Mit welchen Funktionen steht das limbische System in Verbindung?
Emotionen, dazu gehört aber auch die Geruchswahrnehmung, emotional motiviertes
Verhalten...
Aus welchen Strukturen besteht das limbische System?
Septum, Gyrus cinguli, Hippocampus, Amygdala, Fornix, Corpus Mammilariae, Tractus
olfactorius
Welche Strukturen gehören (in einer Reihe) zum Papezschen Kreis?
Hippocampus, Fornix, Hypothalamus, Anteriorer nucleus des Thalamus, Gyrus Cinguli
Welche Strukturen gehören zu einem erweiterten Papezschen Kreis?
Hippocampus, Fornix, Hypothalamus (hauptsächlich Mammilarkörper desselben), Thalamus
(hauptsächlich anteriorer Kern) vielfältige Projektionen in Amygdala, Hypothalamus,
Assoziationscortex, Gyrus Cinguli, Cingulum
Was wissen sie über Phines Gage?
Er war ein Bahnarbeiter dem ein Eisenstecken quer durchs Gehirn stieß, als er ihn in den
Boden schlagen wollte. Er überlebte diesen Unfall, hatte aber einen völlig veränderten
Charakter. Dies lag nicht am traumatischen Erlebnis selbst sondern an der Läsion im linken
Bereich des Cortex, wo vor allem der orbitale Bereich und das Limbische System betroffen
waren. Er war, so sagten seine Freunde, „no longer Gage“, denn er war nun launisch,
respektlos, pietätlos, unentschlossen, ordinär...
Was versteht man unter frontaler Lobotomie, unter welchen Voraussetzungen wurde sie
durchgeführt und was waren die Effekte?
Vor 100 Jahren wurde diese Operation durchgeführt, um nicht passendes Verhalten zu
verändern, z.B. Phobiker oder Depressive wurden behandelt, indem ihnen mit einer nadel
durch das Auge in den präfrontalen Cortex gefahren und dort durch Bewegungen dieser
zerstört wurde. Zwar waren die Patienten nun vom Zwangsverhalten befreit doch hatten dafür
Plusvarianten (Verbal- und Handlungsagressionen) und Minusvarianten (Verlust sozialer
Umgangsformen, Aufmerksamkeitsdefizite, Defizite exekutiver Funktionen) und litten z.B. an
dem Phänomen der Konfabulation. Das alles, ohne dass der IQ dadurch beeinträchtigt wäre.
Was versteht man unter Klüver-Bucy Syndrom, wann tritt es auf und warum kommt es zu den
entsprechenden Ausfällen?
Dieses Syndrom tritt auf, wenn beide Temporallappen mit den darin enthaltenen Amygdalas
(also beider Hemisphären) entfernt bzw. .läsioniert sind: bilaterale temporale lobectomie.
Die Ausfälle sind die Psychic blindness, Abnahme von Furcht, Emotionsausdruck von
Gesichtern können nicht mehr erkannt werden und das klassische Konditionieren funktioniert
nicht mehr. Bei Tieren wurde zudem eine Orale Tendenz, Hypermetamorphosis und
Veränderungen des Sexualtriebes erkannt.
Welche Kerngruppen sind in der Amygdala zu unterscheiden?
Corticomedialer, Centraler und Basolateraler Nucleus
Welche Kerngruppe ist mit „Mandelkern“ gemeint?
Amygdala
Woher erhält der basolaterale Teil der Amygdala einen Grossteil seiner Information?
visuelle, auditive, gustatorische und taktile afferenzen
Welche Ausfallserscheinungen hatte die Patientin S.M, die eine bilaterale
Amygdalaschädigung aufwies?
Sie konnte den Emotionsausdruck von Gesichtern nicht mehr erkennen.
Wozu führt eine elektrische Reizung der Amygdala?
Im corticomedialen nucleus führt die eR zu predatory agression: Teleonomisch, d.h. dient der
Selbsterhaltung z.B. Jagdagression von Raubtieren
Zeichnen sie in die gegebene Graphik (Bear 459-Amygdala) die Kerngruppenbezeichnung
sowie Input, Output und die weiterführenden Strukturen ein.
oberer Teil – dünne Oberschicht - ist der Corticomediale Nucleus, unterer Teil ist der
Basolaterale Nucleus, kleiner Fleck außen ist der centrale Nucleus
Rest der Frage nicht besprochen?
Zeichnen sie in die gegebene Graphik (etwa Bear 598) die Pfade for predatory (und/oder)
affective aggression ein.
nicht besprochen?
Welche Ergebnisse von Karl Pribram (die in der LV besprochen wurden) über bilaterale
Läsionen der Amygdala bei Affen kennen sie?
Die Tiere konnten aggressives Verhalten ihrer Kollegen nicht mehr deuten und verloren ihren
Status innerhalb der Gruppe.
wie vorher: Orale tendenz, Hypermetamorphose, sexual changes (gesteigerter Sexualtrieb,
Masturbation...), emotional changes (abnahme von angst)
Nennen Sie 4 Strukturen, in denen eine selbstinduzierte elektrische Reizung zu weiterer selbst
durchgeführter Stimulation führt.
Substanzia nigra, Septale Region, Lateraler Hypotalamus, Ventral tegmentale Region, Dorsale
Pons
Welches Transmittersystem/Gehirnbereiche ist in den Experimenten (die Reinforcement
zeigten) von Olds & Milner stimuliert worden ?
Dopaminerges System, Pleasure Circuit
Memory
Was versteht man unter deklarativem bzw. non deklarativem Gedächtnis?
Deklaratives Gedächtnis bezieht sich auf Fakten, Events, d.h. ist explizit.
non deklaratives G bezieht sich auf prozedurales Wissen, d.h. ist implizit
In welche Bereiche lässt sich das non-deklaraitive Gedächtnis unterteilen und welche
Strukturen sind dafür jeweils zuständig?
Skills&habits im Striatum,
Klassisches konditionieren der skeletalen Muskulatur über das Cerebellum,
Emotionale Antworten über die Amygdala
Was versteht man unter Hebbschen Cell assemblies?
Gedächtnisinhalte sind in einer größeren Anzahl von Neuronen codiert. Wenn alle feuern ist
ihr Inhalt aktiviert. Beim Lernen verstärken sich die Verbindungen zwischen diesen Zellen,
sodass auch wenn nur ein Bereich aktiviert wird, wird der Rest automatisch angesprochen.
Was wissen sie über den klassischen Patienten von Brenda Millner H.M.?
H.M. hatte epileptische Anfälle und Millner behandelte diese, indem sie den Hyppocampus
herausschnitt (sie vermutete die Ursache eben im Temporallappen). H.M. konnte nun aber
keine neue Informationen mehr lernen. Sie merkte sich keine Personen, Orte (auch nicht, wie
er dort hin kam, wo er gerade war), Tätigkeiten etc. Das prozedurale Gedächtnis jedoch
funktionierte noch: Fähigkeiten wie z.B. in Spiegelschrift zu schreiben waren noch erlernbar.
Also war nur die deklarative Gedächtnisgenerierung verloren.
Welche Gehirnstruktur ist grundlegend wichtig für die Gedächtniskonsolidierung?
Der Cortex
Was versteht man unter Place cells, wann sind diese aktiv und wo sind diese zu finden?
Platzzellen befinden sich im Hippocampus und sie feuern immer, wenn sich das Individuum
an einem bestimmten Ort aufhält, d.h. sie sind Positionsspezifisch – aber nicht auf lange Zeit,
sie orientieren sich immer wieder neu.
Welche Probleme treten bei einer Schädigung des präfrontalen Cortex auf?
Arbeitsgedächtnisaufgaben werden durch solche Schädigungen unmöglich. z.B. das Erkennen
eines Strategiewechsels bei einer Tätigkeit kommt nicht mehr zustande.
Nennen Sie einen klassischen Test zur Überprüfung der präfrontal lokalisierten Fähigkeiten.
Wisconsin card sorting test
Was versteht man unter Long Term Potentiation (LTP)?
Langzeitverstärkung der Verbindungen zwischen Neuronen.
Was verändert sich bei der Long Term Potentiation (LTP)?
Die AMPA Rezeptoren an der Postsynapse vermehren sich, sodass die Verschaltung verstärkt
ist.
In welchen Gehirnbereichen wurde LTP hauptsächlich untersucht?
Hippocampus...? Cortex...?
Welche Rolle spielen die NMDA/AMPA Rezeptoren bei der LTP?
Glutamat bewirkt den Natriumeinstrom ins Zellinnere sodass die Zelle depolarisiert. NMDA
Rezeptoren sind normal durch ein Magnesiummolekül blockiert, welches durch die
Depolarisation gelöst wird. NMDA Rezeptoren regulieren die Ein- und Ausströme: Nun
strömt Calzium in die Zelle und führt (über div. andere Rezeptoren und deren Interaktion)
dazu, dass schon vorhandene AMPA Rezeptoren an die Membran gebracht werden. Dadurch
können mehr Neurotransmitter rezeptiert werden, die Synapse vergrößert sich und es wird
mehr Glutamat ausgeschüttet (produziert).
Was ist die Funktion dentritischer Spikes?
nicht gemacht...
Sie sind Ziel synaptischer Inputs
NICHT GEMACHT:
Brain and Language
Wie ist das Verhältnis rechts/linkshemisphärischer Sprachdominanz bei Recht/Linkshändern
(lt. Rasmussen)?
Was bedeutet linkshemisphärisch sprachdominant?
Mit welcher Methode kann man die Sprachdominanz eindeutig überprüfen?
Was versteht man unter Wada-Test und wie wird er durchgeführt?
Was ist der funktionelle Unterschied zwischen Broca und Wernicke Zentrum?
Nennen sie die verschiedenen Typen von Aphasien und skizzieren sie ganz kurz die
entsprechenden Probleme.
Welche Stationen (Cortexareale, Kerngruppen Faserbündel ...) sind für das laut
lesen/nachsprechen von Worten relevant?
Wozu führt eine Durchtrennung des fasciculus arcuatus?
Welche anatomischen Unterschiede zwischen sprachdominanter und (sprach)subdominanter
Hemisphäre kennen sie?
Ad Aufmerksamkeit:
Was versteht man unter ARAS?
Wie und warum verändert sich das EEG nach einer Durchtrennung unterhalb oder oberhalb
der ARAS?
Motivation:
Was versteht man unter ob/ob Mäusen?
Was passiert, wenn ob/ob Mäuse mit Leptin behandelt werden?
Was ist Leptin und wovon hängt der Leptinspiegel im Blut ab?
Was versteht man unter lateral hypothalamischem Syndrom?
Was versteht man unter venrtomedial hypothalamischen Syndrom?
Welche Hypotalamusläsionen können die Nahrungsaufnahme in welche Richtung
beeinflussen?
Welche Hypothalamusbereiche stehen mit langfristiger Regulation von Nahrungsaufnahme in
Verbindung?
Über welche drei (besprochenen) Mechanismen wird die Nahrungsaufnahme kurzfristig
reguliert?
Über welche neuronale Bahnen beeinflusst die Dehnung der Magenwand das Essverhalten?
Über welche neuronale Bahnen beeinflusst CCK das Essverhalten?
Über welche neuronale Mechanismen beeinflusst Insulin das Essverhalten?
Wo wird Insulin produziert und was wird durch den Insulinspiegel reguliert?
Was passiert bei zu hohem, bzw. zu niedrigem Glucosespiegel im Blut?
Zu welcher „Empfindung“ führt ein gemeinsamer hoher Insulin und Blutglucosespiegel?
Wodurch kommt es zum Insulinschock?
Inwiefern ist Serotonin für Nahrungsaufnahme relevant?
Wie verändert sich der Serotoninspiegel im Hypothalamus bei und nach dem Essen?
Auf welches Transmittersystem und auf welche Kerngruppen wirken Cocain, Heroin und
Nicotin am stärksten?
Zeichnen sie in den gegebenen Schnitt (Box 16.3 S 541) die entsprechenden Drogen,
Kerngruppenbezeichnungen und Exzitation/Inhibition ein.
Herunterladen